2 - Cardiology

Editors: Schrier, Robert W.

Title: Internal Medicine Casebook, The: Real Patients, Real Answers, 3rd Edition

Copyright 2007 Lippincott Williams & Wilkins

> Table of Contents > Chapter 2 - Cardiology

function show_scrollbar() {}

Chapter 2

Cardiology

Simon Shakar

Ronald Zolty

Joann Lindenfeld

Acute Pericarditis and Cardiac Tamponade

  • What are the most common causes of acute pericarditis?

  • What is cardiac tamponade?

  • Does acute pericarditis often result in cardiac tamponade?

  • What are the signs and symptoms of pericarditis and tamponade?

  • How is the echocardiogram helpful in the diagnosis of pericarditis or tamponade?

  • What is the treatment for cardiac tamponade?

P.19


Discussion

  • What are the most common causes of acute pericarditis?

    The most common causes of acute pericarditis are idiopathic, viral infection, uremia, myocardial infarction MI), trauma, cardiac surgery, and neoplasm.

  • What is cardiac tamponade?

    Cardiac tamponade results from accumulation of fluid within the pericardium. As fluid accumulates, intrapericardial pressure increases, limiting filling of the heart and reducing stroke volume. As intrapericardial pressure rises, cardiac filling is increasingly limited. Ultimately, pressures equalize in the left atrium, pulmonary vasculature, right atrium, and superior vena cava (SVC); ventricular filling is progressively impaired and circulatory collapse ensues.

  • Does acute pericarditis often result in cardiac tamponade?

    Acute pericarditis results in tamponade only rarely. Tamponade is more common in end-stage renal disease and neoplastic disease despite the frequent absence of an identifiable episode of acute pericarditis in these conditions.

  • What are the signs and symptoms of pericarditis and tamponade?

    The most common symptom of acute pericarditis is chest pain. The pain is generally sharp and is worse with cough, deep inspiration, and recumbency. A pericardial friction rub is the most common finding in acute pericarditis. It often has three components that occur in systole, and early and late diastole when the heart is moving and the pericardial surfaces rub against one another. Symptoms of tamponade depend on the degree of hemodynamic compromise. The common symptoms of pericardial effusion with tamponade include dyspnea (80%), cough (30%), orthopnea (25%), and chest pain (20%). The common signs of pericardial effusion with tamponade are jugular venous distension and tachycardia (both nearly 100%), pulsus paradoxus (89%), systolic blood pressure 90 mm Hg (52%), and pericardial rub (22%).

  • How is the echocardiogram helpful in the diagnosis of pericarditis or tamponade?

    The echocardiogram is the most accurate and easily available tool to detect and quantify pericardial fluid. However, it is often not of diagnostic value in acute pericarditis because the absence of pericardial fluid does not exclude the diagnosis of acute pericarditis, especially in idiopathic or viral pericarditis. In patients with pericarditis due to neoplasm, bacterial infection, trauma, or cardiac surgery, the echocardiogram may provide helpful information about the etiology of the effusion. For example, metastases may be visible on the pericardial surfaces.

    The echocardiogram is the most commonly used technique for the diagnosis of cardiac tamponade. Typical findings in addition to the presence of pericardial fluid include right atrial and right ventricular diastolic collapse, exaggerated respiratory changes in tricuspid and mitral valve flow, and plethora of the inferior vena cava. Because the limitation of cardiac filling is progressive as the effusion increases, findings of tamponade may be detected by

    P.20


    echocardiogram before the classically described clinical triad of hypotension, paradoxical pulse, and increased systemic venous pressure.

  • What is the treatment for cardiac tamponade?

    Cardiac tamponade requires immediate treatment to relieve the increased end-diastolic pressure and inadequate ventricular filling. The treatment of cardiac tamponade consists of withdrawal of fluid from the pericardial space, generally through a needle inserted percutaneously a procedure called pericardiocentesis. Pericardiocentesis may be performed using echocardiographic guidance to place a needle or a catheter in the intrapericardial space or in the cardiac catheterization laboratory using fluoroscopic guidance. Intravenous (IV) fluids such as blood or saline may be used, but only as a temporizing measure. Volume administration is useful only in hypovolemic patients. In normovolemic patients, the administration of fluid may exacerbate the intrapericardial pressure.

Case

A 78-year-old man with a past history remarkable only for gout is seen because of the acute onset of chest pain. He describes a 4-day prodrome of rhinorrhea, nonproductive cough, myalgias, and anorexia. Approximately 8 hours before he is seen in the emergency room (ER), he began to notice the gradual onset of sharp substernal chest pain, worse with inspiration, relieved by sitting up, and associated with diaphoresis.

The pain is slightly worse with exertion but is not relieved by sublingual nitroglycerin (NTG) administered in the ER, although morphine sulfate and oxygen do seem to alleviate his discomfort. His temperature is 101 F (38.5 C), his heart rate is 105 beats per minute and regular, his respiratory rate is 17 per minute, and his blood pressure is 105/65 mm Hg. The remainder of the physical examination is normal. The electrocardiogram (ECG) is interpreted by the ER staff to show sinus tachycardia with ST-segment elevations inferiorly and nonspecific ST- and T-wave changes elsewhere. An arterial blood gas determination performed on room air shows normal arterial oxygenation. The chest radiographic study is normal.

The ER staff starts an IV heparin drip and a platelet glycoprotein IIb-IIIa inhibitor infusion for the treatment of a presumed acute coronary syndrome (ACS). An IV NTG infusion and oxygen therapy are instituted but, despite these measures, the pain continues. The cardiac catheterization team is called to consider coronary angiography. Antacid therapy does not relieve the pain and only morphine sulfate seems to offer relief. Blood tests reveal a normal troponin, normal electrolytes, normal D-dimer, and normal renal function. The hemoglobin is normal but the white blood cell count is mildly elevated.

The patient is taken to the catheterization laboratory and his coronary angiogram reveals diffuse, mild, nonobstructive coronary artery disease (CAD). The IIb-IIIa inhibitor is discontinued. When the patient is transferred to the coronary care unit, the ECG shows continued evolution with ST-segment elevations of less than 2 mm in leads I, II, III, aVL, aVF, and V2 to V6 that do not respond to IV NTG. The patient's chest pain persists.

Further increments of NTG are given in an IV infusion and the patient's blood pressure begins to decrease. After 2 hours, the patient continues to writhe in pain, complains of feeling dizzy and having a severe headache, and vomits after the fifth dose of IV morphine

P.21


sulfate. You are asked to see the patient and your examination reveals sinus tachycardia, a blood pressure of 82/50 mm Hg (no pulsus paradoxus), a respiratory rate of 16 per minute, a temperature of 101 F (38.5 C), clear lung fields, and no elevation in the jugular venous pressure, but a three-component pericardial friction rub is heard over the precordium. The hemoglobin level is stable.

  • What is the most likely clinical diagnosis of this patient's chest pain?

  • On the basis of your clinical impression of this patient's presentation, what features would be expected on the ECG?

  • Is a normal troponin helpful in acute MI?

  • What is the most effective treatment for acute pericarditis?

  • What is the most likely cause of the hypotension in this patient

Case Discussion

  • What is the most likely clinical diagnosis of this patient's chest pain?

    The most likely clinical diagnosis of this patient's chest pain is acute idiopathic or viral pericarditis. Relatively common causes of acute chest pain that must be considered are MI or ACS, pericarditis, aortic dissection, pneumonia, pulmonary embolus, costochondritis, and pneumothorax. The pertinent features of the history and physical examination that lead to this diagnosis are that the pain was preceded by a viral prodrome and was very clearly positional and exacerbated by inspiration, which strongly suggests pericardial pain. Pericardial pain does not improve with NTG, but the lack of response to NTG does not exclude an acute MI. The patient's vital signs were stable except for a slight fever and tachycardia that are also very frequent in either acute pericarditis or MI. The absence of tachypnea, together with the normal examination findings and normal D-dimer, make acute pulmonary embolization unlikely. Acute costochondritis is often positional but associated with exquisite pain on palpation of the involved costochondral junction, and is not associated with ECG changes. If the examination and chest radiographic findings are normal and there is no past history of smoking, forceful coughing, or trauma, the likelihood of acute pneumothorax is low.

    The remaining two diagnoses, acute pericarditis versus MI, can often be differentiated on the basis of the history and physical examination findings, the ECG, and troponin. The sharp quality of the substernal chest pain, which is associated more with the recumbent position, deep breathing, and coughing, and which is improved by sitting up, is atypical for MI but a classic symptom of pericarditis. The ECG was initially more consistent with pericarditis but an acute MI could not be excluded. The absence of significant coronary obstruction strongly argued against an acute MI, a finding confirmed by the normal troponin.

  • On the basis of your clinical impression of this patient's presentation, what features would be expected on the ECG?

    Sinus tachycardia and ST-segment elevation are often the earliest ECG findings, although the absence of ECG changes does not exclude the diagnosis of pericarditis.

    P.22


    The typical changes of acute pericarditis often evolve over hours or days and are thought to be caused by a myocardial current of injury due to inflammation. The ECG in acute pericarditis evolves usually through four stages over several days. There is early diffuse ST-segment elevation in stage 1. This differs from the ST-segment elevation of acute MI, which is usually localized (anterior, inferior, or lateral), with the ST segments convex upward. In pericarditis, the ST-segment elevation is concave upward and usually involves all the leads except aVR and V1. Stage 2 is defined by normalization of the ST segments and stage 3 is characterized by the development of diffuse T-wave inversions. In stage 4, the T waves return to their normal configuration. PR segment depression is also common in the early phases of acute pericarditis even in the absence of ST-segment elevation and is strongly suggestive of acute pericarditis. An important exception is in pericarditis following an acute MI, in which typical ECG changes of pericarditis may not be present or may be atypical.

  • Is a normal troponin helpful in excluding an acute MI?

    A normal troponin 8 or more hours after the onset of chest pain generally excludes acute MI but does not exclude ACS. However, a mildly elevated troponin may be present with acute myopericarditis. Myocarditis is an inflammatory disease of the cardiac muscle, which can be caused by a variety of different illnesses, many of which are infectious. Typically, myocarditis is associated with cardiac enzyme elevation that reflects myocardial necrosis. When chest pain occurs in the setting of myocarditis it may be associated with concomitant pericarditis and is called myopericarditis.

  • What is the most effective treatment for acute pericarditis?

    In the treatment of idiopathic or viral pericarditis, the goals of therapy are relief of pain and resolution of inflammation. First-choice therapy is the administration of nonsteroidal antiinflammatory drugs (NSAIDs) or aspirin. The administration of colchicine alone or in combination with NSAIDs might be another therapeutic alternative. The use of corticosteroids is usually reserved for patients with pericarditis secondary to autoimmune disease.

  • What is the most likely cause of the hypotension in this patient?

    The hypotension in this patient is most likely due to the cumulative effects of the medications he has been given (morphine and NTG). The accumulation and potentiation of medications, especially in the elderly, is a common clinical problem in the acute care setting. The combination of morphine and NTG in this patient may have induced sufficient vasodilation to cause hypotension.

    Bleeding is also a possible cause of the hypotension. The administration of IV heparin, aspirin, and platelet glycoprotein IIb-IIIa inhibitor agents may result in gastrointestinal bleeding and melanotic stools. The absence of jugular venous distention and a paradoxical pulse argues against tamponade, but these findings may be absent with vasodilation or volume depletion. A more worrisome possibility is hemorrhagic pericarditis, especially because a new friction rub is heard. If the hypotension does not resolve quickly with discontinuation of NTG and morphine, an echocardiogram is indicated to exclude cardiac tamponade.

P.23


Suggested Readings

Imazio M, Bobbio M, Cecchi E, etal. Colchicine in addition to conventional therapy for acute pericarditis: results of the COlchicine for acute PEricarditis (COPE) trial. Circulation 2005;112(13):2012 2016.

LeWinter MM, Kabbani S. Pericardial diseases. In: Braunwald E, ed. Heart disease: a textbook of cardiovascular medicine, 7th ed, Philadelphia: WB Saunders, 2005:1757.

Merce J, Sagrista-Sauleda J, Permanyer-Miralda G, etal. Correlation between clinical and Doppler echocardiographic findings in patients with moderate and large pericardial effusion: implications for the diagnosis of cardiac tamponade. Am Heart J 1999;138:759 764.

Spodick DH. Acute cardiac tamponade. N Engl J Med 2003;349:684 690.

Troughton RW, Asher CR, Klein AL. Pericarditis. Lancet 2004;363:717 727.

Acute Pulmonary Edema

  • What are the two most common underlying mechanisms of pulmonary edema?

  • What are the most common causes of acute cardiogenic pulmonary edema?

  • What is the immediate treatment of acute cardiogenic pulmonary edema?

Discussion

  • What are the two most common underlying mechanisms of pulmonary edema?

    Acute pulmonary edema can have a cardiogenic or noncardiogenic etiology. In cardiogenic pulmonary edema, a high pulmonary capillary pressure is responsible for the transudation of protein-poor fluid into the lungs caused by an imbalance of Starling's forces. With acute rises in pulmonary capillary pressure, the pulmonary lymphatics cannot rapidly increase the rate of fluid removal; as a result, pulmonary edema occurs.

    Noncardiogenic pulmonary edema is caused by altered alveolar capillary permeability due to acute lung injury. Transudation of fluid into the alveolar space is not dependent on an elevated pulmonary capillary wedge pressure but is exacerbated by an elevated pulmonary capillary pressure. The disorders most frequently resulting in increased permeability pulmonary edema are the acute respiratory distress syndrome (ARDS) and, less commonly, high altitude and neurogenic pulmonary edema.

  • What are the most common causes of acute cardiogenic pulmonary edema?

    The most common causes of acute cardiogenic pulmonary edema are acute ischemia and accelerated hypertension, both causing a sudden increase in left ventricular end-diastolic pressure. Both etiologies result in a stiff left ventricle and decreased diastolic ventricular compliance, impairing ventricular filling during diastole (diastolic dysfunction). Systolic dysfunction may also occur. Other causes of acute cardiogenic pulmonary edema include acute mitral regurgitation such as might result from acute ischemia or a ruptured chordae

    P.24


    tendinea, or infectious endocarditis, or discontinuation of antihypertensive medications. Acute pulmonary edema may be precipitated by rapid atrial fibrillation or other dysrhythmias. Infection, physical or environmental stresses, changes or noncompliance with medical therapy, dietary indiscretion, or iatrogenic volume overload are less common, but important, causes.

  • What is the immediate treatment of acute cardiogenic pulmonary edema?

    The immediate treatment of acute cardiogenic pulmonary edema should consist of oxygen therapy to maintain an oxygen saturation within the normal range (95% to 98%), noninvasive positive-pressure ventilation if oxygen saturation remains low [i.e., continuous positive airway pressure (CPAP) or bi-level positive airway pressure (BiPAP)], IV diuresis with furosemide or other loop diuretics, IV morphine, and IV vasodilators with NTG, nitroprusside, or angiotensin-converting enzyme (ACE) inhibitors. The patient should be sitting upright unless hypotension is present. If the patient has an ACS, therapy should be dominated by intervention to minimize ischemic injury. If the acute pulmonary edema is associated with shock, IV inotropic drugs such as milrinone or dobutamine may be necessary. If severe hypertension is present, IV nitroprusside or other rapidly acting agents such as labetalol should be given to lower systemic blood pressure. Noninvasive positive-pressure ventilation with CPAP or BiPAP has been shown to reduce the need for invasive mechanical ventilation in patients with acute cardiogenic pulmonary edema and even to reduce mortality compared with standard therapy (oxygen by face mask, diuretics, and nitrates); the same has been shown in a recent meta-analysis study. (see section on Essential Hypertension and Hypertensive Emergencies).

Case

A 65-year-old man with a history of hypertension, diabetes mellitus, and exertional chest pressure is seen in the ER complaining of sudden onset of chest pain and severe dyspnea at rest. He is currently taking enalapril (5 mg twice a day) to control his blood pressure. Physical examination reveals a pale white male in acute respiratory distress, who is anxious and diaphoretic. His blood pressure is 180/100 mm Hg, his apical pulse is 170 beats per minute and irregularly irregular, and his respiratory rate is 40 per minute. Examination of the lungs reveals rales extending two thirds up from the base of the lung fields bilaterally. Examination of the heart reveals a jugular venous pressure of 12 cm of water, a third sound (S3), and a grade 2/6 holosystolic murmur heard at the apex. Arterial blood gas determinations performed on room air show a partial pressure of oxygen of 50 mm Hg, a partial pressure of carbon dioxide of 30 mm Hg, and a pH of 7.48. A chest radiograph shows an enlarged heart and pulmonary edema. The ECG reveals atrial fibrillation with a ventricular response of 170 beats per minute, a loss of R waves, and 4 mm of ST elevation anteriorly findings that are consistent with an acute anterior MI. A diagnosis of acute anterior wall MI complicated by atrial fibrillation and pulmonary edema is made.

  • What is causing the pulmonary edema in this patient?

  • What medical therapy should be used to treat this patient acutely, and why?

P.25


Case Discussion

  • What is causing the pulmonary edema in this patient?

    There are several causes of the pulmonary edema in this patient.

    • MI impairs both the systolic and diastolic function of the left ventricle. A loss of the contractile function of the large anterior wall of the left ventricle (systolic dysfunction) and acute stiffening of the damaged myocardium (diastolic dysfunction) lead to elevated filling pressures of the left ventricle and the left atrium. Elevated pulmonary venous and pulmonary capillary pressures produce an imbalance in the Starling's forces, resulting in the transudation of fluid into the interstitium and then into the alveolar space.

    • Atrial fibrillation with a rapid ventricular response (170 beats per minute) contributes to the pulmonary edema because (a) the loss of atrial systolic contraction impairs left ventricular filling, which further elevates the left atrial pressure; (b) the rapid ventricular rate results in significant shortening of diastolic filling time further impairing filling of the left ventricle; and (c) the rapid ventricular rate increases myocardial oxygen demands, which may increase ischemia, which in turn worsens the pulmonary edema.

    • Hypertension, especially when chronic and poorly controlled, produces a stiff, hypertrophied myocardium causing elevated ventricular filling pressures. In the setting of acute MI, an increase in blood pressure caused by anxiety, pain, a catecholamine surge, and peripheral vasoconstriction augments the afterload against which the already compromised left ventricle has to work. This leads to a further elevation in ventricular filling pressures, and worsens any ischemia and mitral regurgitation already present.

    • Anxiety secondary to the pain and breathlessness is likely to increase the heart rate and blood pressure, thereby contributing to pulmonary edema by increasing the afterload.

    • A systolic murmur in this setting most likely represents mitral regurgitation secondary to ischemia and papillary muscle dysfunction or, less commonly, rupture of papillary muscle, or an acute ventricular septal defect (VSD). Both acute mitral regurgitation and a VSD result in a systolic murmur at the lower left sternal border. When mitral regurgitation is acute and severe, the systolic murmur may be soft and may not be holosystolic because the left atrial pressure increases rapidly in systole decreasing the mitral regurgitation jet and murmur. The murmur of VSD is generally loud, harsh, and holosystolic due to the vibration of the muscular ventricular septum and a high pressure gradient between the left and right ventricles throughout systole.

  • What medical therapy should be used to treat this patient acutely, and why?

    The most important intervention in this patient is acute reperfusion. An acute coronary angiography will define the coronary anatomy. Percutaneous coronary intervention (PCI) with angioplasty and/or coronary stenting will be indicated if there is no papillary muscle rupture or VSD. Cardiac surgery is necessary if either repair of a VSD or mitral valve replacement for papillary muscle rupture is necessary.

    P.26


    There are several components to the acute supportive treatment of this patient's pulmonary edema. The administration of oxygen to maintain arterial oxygen saturation above 90% is important because the alveolar edema interferes with adequate oxygen diffusion. Noninvasive positive-pressure support ventilation is also beneficial and should be used in patients who are still hypoxic despite medical treatment. Morphine (1 to 3 mg at a time in an IV push) diminishes anxiety and decreases central sympathetic outflow, thereby reducing both venous and arterial vasoconstriction, resulting in decreases in ventricular preload and afterload, respectively. Morphine should not be given to patients with diminished sensorium or respiratory drive or hypercapnia because it may precipitate respiratory arrest. Furosemide (20 to 80 mg in a slow IV push) or other loop diuretics cause immediate venodilation, followed by diuresis within approximately 5 to 10 minutes. IV sodium nitroprusside may be used to reduce blood pressure if hypertension is present. NTG, administered as sublingual tablets or by IV drip, relieves the pulmonary edema by producing venodilation and treating acute ischemia. Digoxin may be used to slow the ventricular response to atrial fibrillation. IV diltiazem or a -blocker may be used to reduce the ventricular response if the patient can tolerate a negative inotropic agent.

    Multiple studies comparing NTG to furosemide or morphine sulfate have demonstrated greater efficacy and safety and a faster onset of action for NTG. Although ACE inhibitors are generally considered the cornerstone for treating chronic heart failure (HF), several very small studies have demonstrated good results for treatment of acute pulmonary edema with this class of agent. Nevertheless, ACE inhibitors should be used with extreme caution in patients with hypotension or significantly impaired renal function.

    It has been demonstrated that systemic infusion of nesiritide has beneficial hemodynamic actions but may cause significant hypotension and no significant benefit in clinical outcomes compared with IV NTG. Also, some concerns have been raised that nesiritide may be associated with an increased risk of death and worsening renal function.

Suggested Readings

Annane D, Bellissant E, Pussard E, etal. Placebo-controlled, randomized, double-blind study of intravenous enalaprilat efficacy and safety in acute cardiogenic pulmonary edema. Circulation 1996;94(6):1316 1324.

Beltrame JF, Zeitz CJ, Unger SA, etal. Nitrate therapy is an alternative to furosemide/morphine therapy in the management of acute cardiogenic pulmonary edema. J Card Fail 1998;4:271 279.

Cotter G, Metzkor E, Kaluski E, etal. Randomized trial of high-dose isosorbide dinitrate plus low-dose furosemide versus high-dose furosemide plus low-dose isosorbide dinitrate in severe pulmonary edema. Lancet 1998;351:389 393.

Pierard LA, Lancelotti P. The role of ischemic mitral regurgitation in the pathogenesis of acute pulmonary edema. N Engl J Med 2004;35:1681 1684.

Sackner-Bernstein JD, Kowalski M, Fox M, etal. Short-term risk of death after treatment with nesiritide for decompensated HF: a pooled analysis of randomized controlled trials. JAMA 2005;293:1900 1905.

P.27


Ware LB, Matthay MA. Clinical practice. Acute pulmonary edema. N Engl J Med 2005;353:2788 2796.

Aortic Dissection

  • What is acute aortic dissection?

  • What is the most common cause of aortic dissection in the general population, in men younger than 40 years, and in women younger than 40 years?

  • What is the most sensitive initial diagnostic test for aortic dissection?

  • Where are the most common points of origin for aortic dissections?

Discussion

  • What is acute aortic dissection?

    Acute aortic dissection results from a tear in the aortic intima. Driven by systemic pressure, arterial blood enters the diseased media of the vessel. Within this layer, blood creates a separation plane as it dissects the aorta longitudinally. The area of dissection filled with blood is called the false lumen. The shear forces of the dissecting blood can cause additional intimal tears. As the false lumen fills with blood, it may compress the true lumen, resulting in obstruction of major arteries. Infrequently, dissection can be initiated by hemorrhage into the media without an intimal tear.

  • What is the most common cause of aortic dissection in the general population, in men younger than 40 years, and in women younger than 40 years?

    In the ascending aorta, the most common cause of aortic dissection in the general population is medial degeneration usually associated with aging and hypertension. In the abdominal aorta, atherosclerosis plays a more important role. In men younger than 40 years, the most common cause of dissection is Marfan's syndrome associated with the more typical cystic medial degeneration lesions. In women younger than 40 years, 50% of all dissections occur during pregnancy.

  • What is the most sensitive initial diagnostic test for aortic dissection?

    The sensitivities of transesophageal echocardiography (TEE), magnetic resonance imaging (MRI), and computed tomography (CT) scan for detection of dissection are similar, with TEE probably having a slight advantage. In most cases, the preferred initial modality is CT scanning because of availability, safety, and convenience. If the patient is not stable, TEE should be considered first as it can be performed in a monitored setting where acute medical therapy can be administered.

  • Where are the most common points of origin for aortic dissections?

    The most common point of origin for ascending aortic dissection is within 2 in. of the aortic valve. For descending aortic dissection, this point is at

    P.28


    the ligamentum arteriosum, just beyond the takeoff of the left subclavian artery.

Case

A 63-year-old man with a history of CAD and previous inferior MI has the following cardiac risk factors: 30 years of moderately controlled hypertension, 75 pack-years of tobacco use, type 2 non insulin-dependent diabetes mellitus, and a family history of CAD. His total cholesterol level 6 months before this admission was 260 mg/dL.

The patient has been experiencing his usual exertional angina, which is relieved with NTG and rest, without a change in pattern or character during the month before presentation. At 11:00 a.m. on the day of admission, he was lifting a 50-lb bag of fertilizer when he experienced an acute severe (10/10), tearing left precordial chest pain without radiation, but with diaphoresis, nausea, and lightheadedness. The pain was similar to his angina, but he obtained no relief with NTG (0.4 mg sublingually). He comes to the ER, where the physical examination reveals a right arm blood pressure of 80/40 mm Hg, a pulse rate of 110 per minute, and a respiratory rate of 24 per minute. He is a diaphoretic elderly man who is writhing in bed and complaining of left chest pain, which is now radiating to the throat and interscapular area. The cardiovascular examination reveals a tachycardia. The first (S1) and second (S2) sounds are normal and a fourth sound (S4) is present. There is a grade 3/4 diastolic murmur consistent with aortic insufficiency heard at the second right and left intercostal spaces. Examination of the peripheral pulses reveals a diminished right radial pulse, a normal left radial pulse, and normal femoral pulses.

  • What tests would you do first to establish a working diagnosis?

  • How are aortic dissections classified, what are the causes, and what are the common signs and symptoms?

  • What initial therapy is indicated to stabilize this patient's condition?

  • Because aortic dissection is thought to be present, what imaging techniques should be done to confirm the diagnosis and assist in planning further therapy?

  • What definitive therapy should be instituted?

  • What long-term care is indicated for this patient?

Case Discussion

  • What tests would you do first to establish a working diagnosis?

    The first procedure to perform is a careful physical examination. Your examination in this patient confirms the ER findings, but the blood pressure in the left arm is 190/110 mm Hg, and the right arm blood pressure is still 80/40 mm Hg. The discrepancy in pulse and blood pressure between the right and left arms is strongly suggestive of aortic dissection involving the proximal aortic arch. The finding of aortic insufficiency is consistent with involvement of the proximal ascending aorta. A chest radiograph should also be obtained. It is likely to show a widened mediastinum with aortic knob intimal calcium separated from the adventitial border by 1.2 cm. This calcium sign is defined as a separation that exceeds 1.0 cm, and it is pathognomonic for aortic dissection. An ECG should also be obtained to determine

    P.29


    if there is an acute MI, which may result from occlusion of the coronary artery by the dissection. In this patient, the ECG shows diffuse, nonspecific ST-segment and T-wave changes. On the basis of the history of tearing pain and these findings, the likelihood of aortic dissection is deemed high in this patient.

  • How are aortic dissections classified, what are the causes, and what are the common signs and symptoms?

    Several classifications for aortic dissection have been proposed, but the most commonly used is the following DeBakey classification:

    • Type I: Dissection originating in the ascending aorta, extending to or beyond the aortic arch

    • Type II: Dissection limited to the ascending aorta

    • Type III: Dissection originating in the descending aorta and extending distally down the aorta or, rarely, extending retrograde into the aortic arch and ascending aorta

    Another classification is the Daily or Stanford scheme that is simpler, and as follows:

    • Type A: All dissections involving the ascending aorta, regardless of the site of origin

    • Type B: All dissections not involving the ascending aorta

    DeBakey types I and II and Stanford A both involve the ascending aorta and are termed proximal dissections, and DeBakey III and Stanford B involve the descending aorta and are termed distal dissections.

    The treatment of aortic dissection depends on whether the dissection involves the proximal or distal aorta. The clinical manifestations are determined by involvement of arterial branches of the aorta (the right brachiocephalic artery in this patient), the aortic valve (aortic insufficiency in this patient) or coronary arteries, or both. A dissection that reaches proximally into the pericardial space can cause tamponade. Approximately two thirds of aortic dissections are proximal, whereas one third is distal.

    The etiology of nontraumatic aortic dissection involves degeneration of the collagen and elastin fibers of the media of the aorta, which usually occurs in patients experiencing a chronic arterial stress, such as hypertension. A specific type of medial degeneration called cystic medial necrosis occurs in patients with Marfan's and Ehlers-Danlos syndromes.

    Other predisposing factors for dissection include congenital coarctation of the aorta, bicuspid aortic valve, atherosclerosis, Noonan's and Turner's syndromes, and giant cell arteritis. Direct external trauma as well as intravascular trauma due to arterial catheterization and intraaortic balloon pumps may result in aortic dissection. Aortic trauma during cardiac surgery, especially aortic valve replacement, may rarely result in dissection.

    The incidence of aortic dissection peaks in the sixth and seventh decades. There is a preponderance of male patients with a male-to-female ratio of 2:1.

    The most common symptom at presentation, seen in more than 90% of patients, is sudden onset of severe chest pain that is immediately maximal in intensity. The pain is unbearable, and often described as a sharp, tearing or ripping sensation. This differentiates it from that of an MI, which is frequently crescendo in nature and

    P.30


    pressure-like. The pain can migrate usually following the path of dissection. Anterior chest pain is usually associated with a proximal dissection, whereas an interscapular pain indicates a distal dissection. The differential diagnosis of aortic dissection includes MI or ischemia, a thoracic nondissecting aneurysm, musculoskeletal pain, mediastinal tumors, and pericarditis.

    Other signs and symptoms of acute aortic dissection depend on involvement of major arterial branches or the aortic valve and are more common with proximal dissection. Aortic insufficiency occurs in up to two thirds of all cases of proximal dissection and is due to dilation of the aortic root, hematoma interfering with leaflet coaptation, tearing of the annulus or leaflet, or a combination of these. Aortic insufficiency is the most common cause of HF in these patients. Neurologic deficits can include stroke, paraplegia, or altered consciousness. Other complications include Horner syndrome resulting from superior cervical ganglion compression and left recurrent laryngeal nerve paralysis causing hoarseness. The involvement of major arterial branches can lead to myocardial, mesenteric, or renal infarctions.

    Rupture of an aortic dissection is more common with the proximal type and can cause acute hemopericardium with cardiac tamponade or a left pleural effusion. Rupture into the airways or esophagus can result in hemoptysis or hematemesis.

  • What initial therapy is indicated to stabilize this patient's condition?

    Medical therapy is indicated initially to stop the progression of the dissection. The patient should be admitted to an intensive care unit with hemodynamic monitoring. Medical therapy is aimed at reducing the mean arterial blood pressure and the velocity of the left ventricular ejection (arterial dP/dt) to minimize arterial shear stress.

    Sodium nitroprusside is a direct vasodilator and decreases arterial pressure in a dose-dependent manner. The aim is to reduce systolic blood pressure to 100 to 120 mm Hg as long as there is adequate organ perfusion. Nitroprusside increases dP/dt if used alone and the administration of -blocking agents blunts this effect. If there are no contraindications to -blockers, they should be given intravenously to reach a heart rate of 60 to 80 beats per minute. Esmolol, a short-acting IV -blocker, may be particularly useful because it can be titrated minute-to-minute to reduce heart rate. Labetalol is also a good choice for the treatment of acute aortic dissection because it is both an - and -blocking drug. In patients who have a contraindication to -blockers, calcium channel blockers such as verapamil or diltiazem delivered by IV route could be used to decrease heart rate and blood pressure.

  • Because aortic dissection is thought to be present, what imaging techniques should be done to confirm the diagnosis of aortic dissection and assist in planning further therapy?

    A transthoracic echocardiogram is a quick and noninvasive modality to confirm aortic insufficiency, assess segmental left ventricular systolic function, and assess the proximal aortic root for the presence of dilation. However, it has poor sensitivity especially for distal dissections. In general, TEE, CT, and MRI are the imaging modalities used to detect dissection.

    TEE is much more sensitive for the detection of dissection, likely the most sensitive of the imaging modalities. It is limited, however, in its capability to assess the distal ascending aorta and the proximal arch. It can assess the proximal aorta, the degree

    P.31


    of aortic insufficiency, left ventricular function, the presence of pericardial effusion, and often permits visualization of the proximal coronary arteries. Therefore, it offers a more complete assessment of the disease and its complications. In hemodynamically unstable patients, this test can be quickly performed at the bedside while treatment is being provided concomitantly, making it the procedure of choice in this instance.

    MRI is highly sensitive and specific in assessing these patients and can visualize the entire thoracic aorta in one view. Using gadolinium, the presence of aortic insufficiency as well as involvement of major branch vessels can be assessed in a large number of patients (i.e., the subclavian or carotid artery). This technique cannot be used in patients with pacemakers and defibrillators. MRI scanners limit access to the patient during the test for up to 30 to 40 minutes, which is disadvantageous in unstable patients.

    A contrast CT scan (especially helical CT) is good for defining the extent of an aortic dissection, that is, proximal versus distal. CT angiography can also assess involvement of major aortic branches. Its major advantages are very high sensitivity and availability. A disadvantage is that it rarely defines the site of the intimal tear.

    The previous gold standard for the diagnosis of aortic dissection was aortography. This modality can define the site of the intimal tear, the severity of aortic insufficiency, coronary artery involvement, and the extent of the dissection proximal versus distal. However, aortography has been shown to have a lower sensitivity compared with the other modalities discussed above. Therefore, the current gold standard varies depending on the availability of imaging modalities. A helical CT is available in most institutions and is very accurate. A TEE, especially in unstable patients, has been recommended as the first test by the European Society of Cardiology. MRI is considered by many as the first test to be performed but is not always available.

  • What definitive therapy should be instituted?

    Untreated acute aortic dissection is associated with 25% mortality at 24 hours and a death rate of more than 75% at 1 month. In general, surgical repair is preferred for acute proximal dissection or in distal dissections when vital organ or limb compromise is present, for rapid expansion or formation of a saccular aneurysm, for rupture, in the presence of uncontrolled pain, or in patients with Marfan's syndrome. Medical therapy (reducing the blood pressure and dP/dt) is adequate for uncomplicated acute distal dissections as there is less risk of complications. It is also recommended in chronic (present for >2 weeks) proximal or distal dissections as these patients have survived the period of highest mortality risk.

  • What long-term care is indicated for this patient?

    It is essential to rapidly control the patient's hypertension and decrease the rate of pressure rise in the left ventricle, preferably with blockers. The long-term prognosis in hospital survivors is good, with an actuarial survival rate only slightly worse than that for age-matched subjects. The type of dissection or therapy used does not influence the outcome after discharge from the hospital. The highest risk for recurrent dissection or aneurysm expansion is in the first 2 years. Careful follow-up during this initial period is important to ensure adequate blood pressure control and monitor for recurrence. This would include physical examination and chest x-rays. Serial imaging with CT scanning, TEE, or MRI should also be part of this follow-up.

P.32


Suggested Readings

Hagan PG, Nienaber CA, Isselbacher EM, etal. The international registry of acute aortic dissection (IRAD): new insights into an old disease. 2000;283(7):897 903.

Nienaber CA, Eagle KA. Aortic dissection: new frontiers in diagnosis and management: part I: from etiology to diagnostic strategies. Circulation 2003;108:628 635.

Nienaber CA, Eagle KA. Aortic dissection: new frontiers in diagnosis and management: Part II: therapeutic management and follow up. Circulation 2003;108:772 778.

Sabik JF, Lytle BW, Blackstone EH, etal. Long-term effectiveness of operations for ascending aortic dissections. J Thorac Cardiovasc Surg 2000;119(5):946 962.

Chronic Heart Failure

  • What are the most common underlying diseases causing chronic HF in the U.S. population?

  • Is HF always associated with a decreased ejection fraction (EF)?

  • What is myocardial remodeling and what are its consequences?

  • Which drug classes have been shown to prolong survival in patients with HF?

  • What devices have been shown to prolong survival in patients with HF?

Discussion

  • What are the most common underlying diseases causing chronic HF in the U.S. population?

    In the United States and most developed countries, hypertension and ischemic heart disease are the most common causes of HF. Valvular heart disease and cardiomyopathy are less common causes, but are still frequently encountered.

  • Is HF always associated with a decreased EF?

    Systolic dysfunction is defined as a decrease in contractile function most commonly measured as a decrease in EF. Many patients with HF have a decreased EF. However, almost half of all patients with HF have a normal EF. In some of these patients, diastolic dysfunction is the cause. Diastolic dysfunction results when the heart is stiff and ventricular filling is impaired, resulting in increased end-diastolic pressures. Patients may have diastolic dysfunction with or without systolic dysfunction. Typical signs and symptoms of HF occur with either normal or abnormal EF. Typically, the prevalence of HF with a normal EF is most common in elderly women.

  • What is myocardial remodeling and what are its consequences?

    After myocardial injury with resulting systolic dysfunction, there is often a progressive deterioration in the structure and function of the ventricular myocardium a process termed myocardial remodeling. Myocardial remodeling is characterized by progressive ventricular enlargement and decreasing EF. This progressive remodeling is at least partially responsible for the high

    P.33


    mortality rates in patients with HF. Although the specific molecular and cellular events that lead to remodeling are not entirely understood, many factors that promote remodeling have been described. These mechanisms include increased wall stress and activation of the renin angiotensin and -adrenergic systems. Blockade of these systems would be expected to slow or prevent myocardial remodeling and improve survival in patients with HF and systolic dysfunction.

  • Which drug classes have been shown to prolong survival in patients with HF?

    In patients with HF due to a decreased EF, ACE inhibitors and -adrenergic receptor antagonists -blockers) have been shown to improve survival. Angiotensin-receptor blockers (ARBs) are probably equivalent to ACE inhibitors and may be substituted, especially if there is intolerance to ACE inhibitors due to cough. Either aldosterone antagonists or ARBs also improve survival when added to ACE inhibitors and -blockers, but care must be taken to monitor patients carefully to avoid hyperkalemia; the use of all four drug classes together is not advised for most patients because of the risk of hyperkalemia. Digoxin may be helpful to improve symptoms but does not improve survival. Loop diuretics such as furosemide, bumetanide, and torsemide clearly relieve congestion caused by salt and water retention but have not been shown to improve survival. The combination of hydralazine and isosorbide dinitrate improves survival in African Americans with systolic dysfunction and New York Heart Association (NYHA)'s class III IV HF.

    In patients with HF and normal EF only one major trial has been conducted. This trial using the ARB candesartan did not show a significant benefit on hospitalization or mortality rate. Loop diuretics are valuable in relieving congestive symptoms in these patients but no clinical trials have been conducted.

  • What devices have been shown to prolong survival in patients with HF?

    Patients with significant systolic dysfunction (EF 35%), and NYHA class II-III heart failure have improved survival when an internal cardiac defibrillator (ICD) is implanted. The ICD detects serious ventricular arrhythmias and corrects them either with pacing or a shock. Cardiac resynchronization therapy (CRT) is based on the concept that patients with left ventricular systolic dysfunction often have ventricular dyssynchrony. Dyssynchrony is most often seen when the QRS duration is 120 milliseconds or more and most clinical trials have used this QRS duration as an entry criteria. Dyssynchrony means that the left ventricular contraction is discoordinated, resulting in a lower stroke volume and increased wall stress. By pacing both the ventricular septum (with a pacer in the right ventricular apex) and the lateral wall of the left ventricle (through a pacer advanced through the coronary sinus into a lateral coronary vein) the coordination of ventricular contraction is improved, increasing cardiac output. In patients with an EF of 35% or less and HF, CRT improves symptoms, hospitalizations, and mortality.

P.34


Case

A 42-year-old white man is seen in the ER with a chief complaint of shortness of breath that has lasted for 1 week. He reports having had a viral syndrome approximately 3 weeks before admission. Subsequently, he noted the development of lower extremity edema, a 15-lb weight gain, dyspnea on exertion, and orthopnea. Currently he complains of dyspnea at rest. Physical examination reveals an irregularly irregular heart rate of 130 per minute. His blood pressure is 90/60 mm Hg, and his respiratory rate is 22 per minute. Examination of the jugular venous pressure demonstrates a mean pressure of 12 to 14 cm of water with a prominent V wave. Lung examination reveals bibasilar dullness with rales extending one fourth of the way up from the basal lung fields bilaterally. Cardiac examination findings are significant for a diffuse point of maximal impulse, which is displaced to the anterior axillary line. The S1 and S2 are of variable intensity, and a prominent S3 gallop over the displaced cardiac apex is appreciated. There is a grade 2/6 holosystolic murmur that is heard best at the cardiac apex, with prominent radiation to the axilla and no change with respiration. On examination of the abdomen, an enlarged, tender liver is found. The extremities are cool and exhibit 2+ pitting edema. The ECG shows atrial fibrillation with nonspecific ST-T wave changes, a left bundle branch block (LBBB) and occasional ventricular premature beats. Arterial blood gas measurements performed with the patient on 4 L of oxygen per minute by nasal cannula reveal a pH of 7.46, a PO2 of 52 mm Hg, a PCO2 of 32 mm Hg, and a bicarbonate (HCO3-) concentration of 26 mmol/L.

  • Does this patient have left, right, or biventricular failure?

  • An S3 is heard, but no S4. Why?

  • What chest radiographic findings would you expect to see in this patient?

  • What neurohormonal mechanisms are likely to be activated in this patient?

  • What diagnostic tests should be performed?

  • What treatment options would likely be beneficial in this patient?

  • Is it possible that the ventricular function will improve with medical therapy?

    Your patient improved after diuresis and administering ACE inhibitors and -blockers. Six months later his EF has increased from 20% to 29%. He is on digoxin with therapeutic levels and an aldosterone antagonist with normal serum creatinine and potassium. He has no resting dyspnea or edema, but does have dyspnea with simple tasks.

  • In which NYHA class and American College of Cardiology/American Heart Association (ACC/AHA) stage would you categorize this patient's symptoms?

  • What is this patient's expected mortality rate in his current condition?

Case Discussion

  • Does this patient have left, right, or biventricular failure?

    This patient has findings indicating both right and left ventricular failure (biventricular failure). The cool extremities, tachycardia, and narrow pulse pressure suggest poor forward cardiac output and could reflect either right or left ventricular failure. A left ventricular S3 gallop and pulmonary rales are signs of left ventricular

    P.35


    failure. The bibasilar dullness suggests the presence of bilateral pleural effusions, which may be seen in the setting of either right or left ventricular dysfunction. The apical murmur most likely represents mitral regurgitation because it is loudest at the apex, it radiates to the axilla, and it does not change with respiration. We do not know from the history whether the patient had a preexisting valvular disorder. Secondary mitral or tricuspid regurgitation occurs commonly in patients with ventricular enlargement and dysfunction due to distortion of the supporting structures of the atrioventricular valves. Tricuspid regurgitation causes a large V wave in the jugular venous pulse.

    There are many signs of right ventricular failure in this patient. Elevated central venous pressure is apparent from the patient's jugular venous distention. Kussmaul's sign is the lack of a fall in the jugular venous pressure with inspiration and is due to the right ventricle's inability to handle the augmented venous return. It may be encountered in patients with right ventricular failure or constrictive pericardial disease. The patient's enlarged liver is the result of hepatic congestion stemming from increased back pressure on the hepatic vein. The pitting edema in the lower extremities is caused by elevated hydrostatic pressure in the venous system, resulting in extravasation of fluid into the interstitial space of the ankles, where the forces of gravity are the greatest.

  • An S3 is heard, but no S4. Why?

    An S3 is a low-frequency sound heard 0.13 to 0.16 second after S2. An S3 occurs at the end of the rapid phase of ventricular filling and is most likely due to the vibration of the chordae tendineae or the left ventricular wall with rapid filling, and may arise from the right or left ventricle. A left ventricular S3 is best heard with the bell of the stethoscope at the cardiac apex. A right ventricular S3 is also best heard with the bell, but is most audible at the lower left sternal border or over the epigastrium. An S3 is a normal finding in children or young adults, but in middle-aged or older patients it is usually a sign of volume overload most often due to HF, as it is in this patient.

    An S4 is a presystolic atrial sound (gallop) that is heard when the ventricle is poorly compliant. Given the patient's volume overload, it is likely that both ventricles are poorly compliant. However, the patient is in atrial fibrillation, and therefore there are no effective atrial systoles to give rise to an S4 (rarely, an S4 may be heard even in atrial fibrillation because of the high left atrial pressure and increased flow in late diastole).

  • What chest radiographic findings would you expect to see in this patient?

    The likely findings on a chest radiography stem from the effects of volume overload and elevated pulmonary venous pressure. Cardiomegaly, which is defined as a cardiac-to-thoracic diameter ratio exceeding 0.5, is present in most cases in which there is depressed left ventricular systolic function. Cephalization of the pulmonary blood flow occurs and is evidenced by the enlarged pulmonary vessels in the superior portion of the pulmonary tree. The haziness of the central vasculature is a result of the increased hydrostatic pressure and subsequent transudation of fluid into the tissue surrounding the vessels. Kerley B lines are horizontal, thin, sharp lines that extend inward from the periphery of the lungs. They represent edema formation

    P.36


    within the lungs and hypertrophy of the interlobular septa. Pleural effusions may be found in the setting of right or left ventricular failure. When pulmonary congestion is severe and alveolar edema is present, a butterfly or bat-wing infiltrate may be seen centered over the main pulmonary artery.

  • What neurohormonal mechanisms are likely to be activated in this patient?

    The two neurohormonal mechanisms most likely to be activated in this patient are the renin angiotensin aldosterone system and the adrenergic nervous system. The serum norepinephrine level has been shown to correlate inversely with the EF and patient survival in those with chronic HF. Cardiac adrenergic activation occurs even earlier than systemic adrenergic activation. Other hormones that may be activated include vasopressin, endothelin, and multiple cytokines such as tumor necrosis factor and interleukin 1.

  • What diagnostic tests should be performed?

    Initially a complete blood count, and thyroid stimulating hormone (TSH), electrolyte, renal and hepatic function tests should be obtained to determine if there are electrolyte abnormalities that need to be corrected, if there is significant underlying renal or hepatic disease, and to determine if anemia or thyroid abnormalities may have exacerbated the heart failure. An ECG should be performed to determine if there has been an MI or if arrhythmias are present. A chest x-ray will confirm the HF and detect significant underlying pulmonary problems. An echocardiogram will evaluate ventricular size and function, the presence of valve abnormalities, and may often suggest the underlying etiology of the ventricular dysfunction. For example, if the anterior wall is akinetic and scarred, a previous MI can be inferred. When the left ventricle is large and has global dysfunction, it may be difficult to determine if there is underlying CAD. When the patient has stabilized an exercise, echocardiogram or nuclear study may reveal reversible ischemia. Coronary angiography may be necessary to exclude significant CAD if noninvasive studies do not clearly exclude ischemia. If there is no significant coronary disease and no significant valve disease, the diagnosis is likely idiopathic cardiomyopathy. The type of cardiomyopathy can generally be categorized by echocardiogram as dilated, hypertrophic, or restrictive with dilated cardiomyopathy being the most common. A series of tests, depending on the type of cardiomyopathy, should be carried out to exclude specific etiologies.

  • What treatment options would likely be beneficial in this patient?

    The general goals for the medical treatment of HF are as follows:

    • Identify and treat the underlying condition.

    • Eliminate any precipitating factors.

    • Treat the symptoms.

    • Improve survival.

    The first step is to identify the underlying cause of HF. This may be hypertension, CAD, cardiomyopathy, valvular heart disease, or many other causes. Treatment includes medical treatment for hypertension, coronary angiography and coronary angioplasty or coronary bypass surgery for coronary disease, and valve replacement or repair for valve disease.

    P.37


    In patients with HF, it is important to eliminate precipitating factors (e.g., dietary or medication noncompliance, arrhythmias, anemia). Excess alcohol use may cause a cardiomyopathy, but excess alcohol may also exacerbate HF of any cause.

    Symptomatic improvement is usually achieved by relieving the excess salt and water retention with diuretics and by improving preload and afterload with vasodilators particularly the ACE inhibitors. Loop diuretics such as furosemide, torsemide, or bumetanide are most often used because they are more effective than thiazide diuretics when renal perfusion is decreased. Care must be taken to avoid overdiuresis and to replace potassium and magnesium because hypokalemia and/or hypomagnesemia may promote ventricular arrhythmias. The combination of ACE inhibitors and -adrenergic antagonists ( -blockers) is the cornerstone of therapy for patients with HF due to systolic dysfunction. Many ACE inhibitors are now available (captopril, enalapril, lisinopril, quinapril, ramipril, benazepril, trandolapril, fosinopril, moexipril), and there does not appear to be a clear therapeutic advantage to the use of one over another. However, the target dose of an ACE inhibitor is best determined from the individual agents that have been studied in patients with HF. By decreasing the conversion of angiotensin I to angiotensin II, these drugs reduce preload and afterload, improve symptoms, and prolong survival in patients with systolic dysfunction. Cough is the most common side effect of ACE inhibitors, but cough is also a common symptom of HF. Care should be taken to exclude HF as a cause of the cough before these drugs are discontinued. Hypotension, renal insufficiency, and hyperkalemia are less frequent but serious side effects of the ACE inhibitors. In general, these occur in patients with severe HF and/or preexisting renal insufficiency. In patients with severe HF or intrinsic renal insufficiency, the ACE inhibitors should be started in very low doses, and the blood pressure and serum potassium and creatinine levels must be monitored carefully.

    -Adrenergic blockers have also been shown to improve survival in patients with systolic dysfunction and HF. Although the benefit on symptoms is less clear than with ACE inhibitors, -blockers produce a larger improvement in remodeling, EF, and survival. Because -blockers reduce heart rate and initially decrease contractility, introduction of treatment or up-titration may result in worsening of symptoms. These drugs must therefore be started in low doses and up-titrated slowly, and patients must be monitored carefully. Patients with decompensated HF usually should not be given -blockers. Several -blockers (carvedilol, metoprolol, and bisoprolol) have been shown to reduce mortality in patients with HF. It is not yet clear if there are advantages of one over another.

    There are several possible additions to medical therapy in patients with chronic HF due to systolic dysfunction, who remain symptomatic after maximum tolerable doses of ACE inhibitors and -blockers. The addition of aldosterone antagonists reduces mortality in patients with severe chronic HF and in those who have experienced HF following an MI. ARBs such as losartan, candesartan, irbesartan, and valsartan block the angiotensin II receptor directly. They appear to have beneficial effects in reducing cardiovascular mortality and hospitalization for HF when added to ACE inhibitors and -blockers. Adding both aldosterone antagonists and ARBs to ACE inhibitors is probably not reasonable for most patients because of the increased risk

    P.38


    of hyperkalemia. The combination of hydralazine and isosorbide dinitrate reduces mortality and hospitalizations and improves quality of life in African Americans with severe chronic HF due to systolic dysfunction. The benefits are less clear in non African Americans. Another option is the use of digoxin, which results in an improvement in symptoms and a reduction in hospital admissions but no reduction in mortality. Sodium restriction is an essential part of any program designed to treat patients with HF. Patients should avoid excess salt and water, weigh themselves daily, avoid NSAIDs, and report any increase in symptoms or weight gain promptly to their physicians.

  • Is it possible that ventricular function will improve with medical therapy?

    This patient has an EF of 20%. ACE inhibitors help prevent further deterioration in EF. -Blockers, if up-titrated to recommended doses, are likely to improve this individual's EF by 7% to 10%. The full improvement may not be seen for up to 6 months. Your patient improved after diuresis and administering ACE inhibitors and -blockers. Six months later his EF has increased from 20% to 29%. He is on digoxin with therapeutic levels and an aldosterone antagonist with normal serum creatinine and potassium. He has no resting dyspnea or edema, but does have dyspnea with simple tasks.

  • In which NYHA class and AHA/ACC stage would you categorize this patient's symptoms?

    This patient continues to have symptoms of NYHA class III HF.

    The four categories that make up the NYHA classification, and their definitions, are:

    • Class I: No symptoms with any level of exercise

    • Class II: Symptoms on more than ordinary activity

    • Class III: Symptoms on activities of daily living

    • Class IV: Symptoms at rest

    The AHA/ACC stages of HF are:

    • Stage A: Risk factors for HF

    • Stage B: Structural heart disease, but no HF

    • Stage C: Structural heart disease and HF

    • Stage D: Structural heart disease and refractory HF.

    Your patient was initially in NYHA class IV and AHA/ACC stage C D. He has improved to NYHA class III, stage C.

  • What is this patient's expected mortality rate in his current condition?

    His expected mortality rate in NYHA class IV HF, if untreated, was 25% to 50% in 1 year. With good medical therapy and internal cardiac defibrillator (ICD)-biventricular pacing device, his yearly mortality may improve to as low as 8% per year. An ICD can prolong survival in patients with HF and systolic dysfunction and should be implanted at this point as medical therapy is unlikely to cause significant additional gains in left ventricular function. Biventricular pacing (implanted in conjunction with the ICD) is indicated in this patient who has systolic dysfunction, ongoing HF symptoms, and a QRS duration greater than 120 milliseconds. Biventricular pacing is likely to improve ventricular function, symptoms, and survival.

P.39


Suggested Readings

Adams KF, Lindenfeld J, Arnold JMO, etal. Executive summary: HFSA 2006 comprehensive HF practice guidelines. J Card Fail 2006;12:10 38.

Bristow MR. Beta-adrenergic receptor blockade in chronic HF. Circulation 2000;101:558 569.

Cleland JG, Daubert JC, Erdmann E, etal. The effect of cardiac resynchronization on morbidity and mortality in HF. N Engl J Med 2005;352:1539 1549.

Francis GS, Tang WH. Pathology of congestive HF. Rev Cadiovasc Med 2003;4(Suppl 2):S14 S20.

Hunt SA, Abraham WT, Chin MH, etal. ACC/AHA 2005 guideline update for the diagnosis and management of chronic HF in the adult. Circulation 2005;112:e154 e235.

Jessup M, Brozena S. Heart failure. N Engl J Med 2003;348:2007 2018.

McClellan MB, Loeb JM, Clancy CM, etal. Angiotensin-converting enzyme inhibitors and angiotensin-receptor blockers in chronic HF. Ann Intern Med 2005;142:386 387.

Essential Hypertension and Hypertensive Emergencies

  • What is the estimated prevalence of systemic hypertension in the U.S. population?

  • What is the most common cause of systemic hypertension?

  • How is hypertension classified?

  • What is the natural history of untreated hypertension?

  • Does medical therapy improve outcomes in hypertension?

  • What is a hypertensive crisis?

Discussion

  • What is the estimated prevalence of systemic hypertension in the U.S. population?

    Hypertension in the United States affects approximately 65 million Americans. However, the prevalence increases with age, so that more than 60% of the population older than 70 years has hypertension. The Framingham Heart Study has demonstrated that 55-year-old normotensive individuals have a 90% lifetime risk of developing hypertension. The incidence of hypertension and its severity is greater in blacks than whites in every age-group beyond adolescence.

  • What is the most common cause of systemic hypertension?

    No cause is found for approximately 90% of patients with hypertension. These patients are said to have essential hypertension. Although the mechanism of essential hypertension is unknown, there are apparently both genetic and environmental factors.

  • How is hypertension classified?

    Normal blood pressure is less than 120/80 mm Hg. Blood pressures of 130 to 139 mm Hg systolic and 80 to 89 mm Hg diastolic are considered

    P.40


    prehypertension. Individuals with prehypertension have twice the lifetime risk of developing hypertension as their normotensive counterparts. Stage 1 hypertension is defined as a systolic blood pressure of 140 to 159 mm Hg and a diastolic blood pressure of 90 to 99 mm Hg, whereas stage 2 hypertension is a systolic blood pressure of 160 mm Hg and greater and a diastolic blood pressure of 100 mm Hg and greater (Table 2-1).

  • What is the natural history of untreated hypertension?

    Uncomplicated hypertension often remains asymptomatic for 10 to 20 years or more. However, there is a direct relationship between the levels of both systolic and diastolic blood pressures and the incidence of stroke, CAD, and HF. Indeed, for every 20 and 10 mm Hg increment in systolic and diastolic pressure respectively, individuals aged 40 to 70 years have a doubling of cardiovascular risk from blood pressures of 115/75 to 185/115 mm Hg. The overall risk of premature cardiovascular disease increases substantially when additional cardiovascular risk factors are present. In fact, the likelihood of a vascular event over the next 10 years can be estimated for any patient on the basis of their age, sex, and other risks (American Heart Association's Coronary and Stroke Risk Handbook). If patients with hypertension are not treated, approximately 50% die of coronary disease, 33% of stroke, and 10% to 15% of renal failure.

  • Does medical therapy improve outcomes in hypertension?

    Clinical trials of antihypertensive therapy have demonstrated an average mean reduction of 40% for stroke, 50% for HF, and 20% to 25% for MI.

  • What is a hypertensive crisis?

    A hypertensive crisis is an acute life-threatening complication of accelerated hypertension. In patients with chronic hypertension and hypertensive crisis, the blood pressure is generally 180/120 mm Hg or greater, but may be lower in patients whose blood pressure was previously normal (e.g., eclampsia). Malignant hypertension is present when there are retinal hemorrhages, exudates or papilledema, and/or malignant nephrosclerosis. When there are signs of cerebral edema, hypertensive encephalopathy is said to be present. Examples of hypertensive crises include:

    • Accelerated/malignant hypertension

    • Hypertensive encephalopathy

    • Atherothrombotic cerebral infarction with severe hypertension

    • Aortic dissection

    • Acute pulmonary edema or left ventricular failure

    • Acute MI

    • Eclampsia

    • Drug-induced hypertension (cocaine)

Case

A 45-year-old African-American man is seen in the outpatient department complaining of intermittent throbbing headaches that have occurred every morning for 2 weeks. He has a history of untreated, asymptomatic, sustained high blood pressure (160 to

P.41


P.42


170/100 mm Hg) of 4 years' duration. He has no history of palpitations, sweating, tremor, or periodic paralysis. His father was also hypertensive and died of stroke at 67 years. The patient has smoked cigarettes, two packs per day, for 30 years. He is taking no medications.

Table 2-1 Classification and Management of Blood Pressure for Adults Aged 18 Years or Older

        Managementa
          Initial Drug Therapy
BP Classification Systolic BP (mm Hg)a   Diastolic BP (mm Hg)a Lifestyle Modification Without Compelling Indications With Compelling Indicationsb
Normal <120 and <80 Encourage    
Prehypertension 120-139 or 80 -89 Yes No antihypertensive drug indicated Drug(s) for the compelling indicationsc
Stage 1 hypertension 140-159 or 90-99 Yes Thiazide-type diuretics for most; may consider ACE inhibitor, ARB, -blocker, CCB, or combination Drug(s) for the compelling indications
            Other antihypertensive drugs (diuretics, ACE inhibitor, ARB, -blocker, CCB) as needed
Stage 2 hypertension 160 or 100 Yes Two-drug combination for most (usually thiazide-type diuretic and ACE inhibitor or ARB or -blocker or CCB)d Drug(s) for the compelling indications
            Other antihypertensive drugs (diuretics, ACE inhibitor, ARB, -blocker, CCB) as needed
aTreatment determined by highest BP category.
bSee reference below.
cTreat patients with chronic kidney disease or diabetes to BP goal of <130/80 mm Hg.
dInitial combined therapy should be used cautiously in those at risk for orthostatic hypotension.
BP, blood pressure; ACE, angiotensin-converting enzyme; ARB, angiotensin-receptor blocker; CCB, calcium channel blocker.
From Chobanian AV, Bakris GL, Black GL, et al. JAMA 2003;289:2560-2572.

His physical examination reveals a blood pressure of 170/110 mm Hg and a heart rate of 90 per minute and regular. His weight is 244 lb and he is 5 ft 10 in. tall. Fundus examination reveals the presence of arterial vasoconstriction. Cardiac examination reveals a laterally displaced sustained point of maximal impulse, S4, no S3, and no murmur. During abdominal examination, no bruit or mass is found and the neurologic and other systems are unremarkable.

  • How should blood pressure be measured?

  • What is the most likely cause of this patient's hypertension?

  • What laboratory tests are indicated?

  • Will lifestyle changes improve his blood pressure?

  • Is drug therapy indicated at this time?

  • What is the target blood pressure with treatment?

Case Discussion

  • How should blood pressure be measured?

    The patient should be seated in a chair with his feet on the floor in a quiet place for at least 5 minutes. At least two measurements should be made with a calibrated instrument. Systolic blood pressure is defined as the blood pressure at which the first sound is heard and diastolic pressure is defined as the pressure at the disappearance of the sounds.

  • What is the most likely cause of this patient's hypertension?

    The most likely cause of this patient's hypertension is essential hypertension.

  • What laboratory tests are indicated?

    The laboratory investigation should include chest radiography (normal), ECG (sinus rhythm with increased voltage but no ST-T wave changes), urinalysis, hematocrit, calcium (all normal), and measurement of the fasting blood sugar (normal), blood urea nitrogen, serum creatinine, electrolytes (normal), cholesterol [total and low-density lipoprotein (LDL) and high-density lipoprotein (HDL)], and triglyceride (total cholesterol is 240 with LDL of 170, HDL of 40, and normal triglycerides).

  • Will lifestyle changes improve his blood pressure?

    Lifestyle modifications are an important part of blood pressure management. Beneficial lifestyle modifications include weight reduction in overweight or obese people, regular exercise, adoption of theDASH (Dietary Approaches to Stop Hypertension) diet which is high in calcium and potassium, limitation of sodium intake, and moderation of alcohol consumption.

  • Is drug therapy indicated at this time?

    Medical therapy is indicated in this patient who has demonstrated stage 2 hypertension (Table 2-1 and Fig. 2-1). The plan of management should commence

    P.43


    with instruction in lifestyle changes and oral antihypertensive drugs with the aim of maintaining blood pressure at less than 140/90 mm Hg. Eliminating coexisting cardiovascular risk factors (especially smoking) and treating the elevated cholesterol will not lower the blood pressure but will lower his risk of subsequent cardiovascular events.

    Figure 2-1 Algorithm for treatment of hypertension. BP, blood pressure; ACE, angiotensin-converting enzyme; ARB, angiotensin-receptor blocker; CCB, calcium channel blocker. (From

    Chobanian AV, Bakris GL, Black GL, etal. The seventh report of the Joint National Committee on prevention, detection, evaluation, and treatment of high blood pressure. JAMA 2003;289:2560 2572

    .)

  • What is the target blood pressure with treatment?

    The target blood pressure with treatment is less than 140/90 mm Hg. If the patient had diabetes or chronic kidney disease the recommended target blood pressure would be less than 130/80 mm Hg.

    You instruct your patient in lifestyle changes and start him on lisinopril 10 mg once daily. In 2 weeks you increase the lisinopril to 20 mg daily because the blood pressure is still 160/98 mm Hg. Electrolytes and creatinine levels are unchanged. The increased lisinopril does not significantly alter the pressure and you add chlorthalidone at 25 mg daily. In 4 weeks his blood pressure is 139/88 mm Hg

    P.44


    and his electrolytes and creatinine levels are normal. The blood pressure remains well controlled for the next 6 months, but the patient does not return for the next follow-up visit and does not respond to phone calls. Two years later, he presents to the ER complaining of blurred vision and severe headaches. His physical examination at that time reveals a blood pressure of 240/140 mm Hg and heart rate of 100 per minute. He is mildly confused and the fundus examination reveals retinal hemorrhages, exudates, and papilledema. Heart examination shows clear lungs and a sustained left ventricular apical impulse and S4. The chest radiograph shows mild to moderate cardiomegaly. His serum creatinine level is 2.4 mg/dL. The ECG shows normal sinus rhythm with increased voltage and ST-segment depression and T-wave inversion. Troponin is normal and he has no chest pain.

    • What is the diagnosis?

    • What is the most likely reason for the fundus findings and the serum creatinine level of 2.4 mg/dL?

    • What should be the plan of treatment now?

    • What is the diagnosis?

      The diagnosis is hypertensive crisis and accelerated malignant hypertension. A hypertensive crisis is considered a medical emergency. Such high blood pressure can cause immediate vascular damage, as seen in this patient. The presence of severe hypertension (diastolic blood pressure of 115 mm Hg or greater) in conjunction with grade 3 (retinal hemorrhage and exudate) or grade 4 (papilledema) funduscopic changes is defined as accelerated or malignant hypertension.

    • What is the most likely reason for the fundus findings and the serum creatinine level of 2.4 mg/dL?

      Modest increases in blood pressure result in arteriolar vasoconstriction. The vasoconstriction keeps tissue perfusion constant. However, with the marked increase in blood pressure causing a sudden increase in tissue perfusion, there is damage to the vascular endothelium causing fibrinoid necrosis in the vessels of the eye and in the kidney. These changes are exacerbated by activation of the renin angiotensin system.

    • What should be the plan of treatment now?

      The patient should be admitted to a monitored unit. An ECG, chest x-ray, electrolytes, urinalysis, hematocrit, and troponin should be obtained. Because the papilledema is consistent with the presence of severe hypertension and represents early brain edema, which may compromise the autoregulation of cerebral blood flow, the treatment approach should be to start him on parenteral antihypertensive drug therapy. The goal of treatment should be to decrease the blood pressure by 20% to 25% maintaining the diastolic blood pressure between 100 and 110 mm Hg or the mean arterial pressure at not less than 120 mm Hg, because an abrupt decrease in the blood pressure to normal levels may produce hypoperfusion to the brain, heart, and kidney due to lack of autoregulation.

    P.45


    A list of parenteral agents used to treat hypertensive emergencies is given in Table 2-2.

    Table 2-2 Parenteral Agents Used to Treat Hypertensive Emergencies

    Drug Route Onset Duration Dose or Dosage
    Vasodilators        
    Sodium nitroprusside IV infusion Seconds-1 min 3-5 min 0.25-10 g/kg/min
    Nicardipine hydrochloride IV 5-10 min 1-4 hr 2-10 mg/hr
    Fenoldopam mesylate IV infusion <5 min 30 min 0.1-0 .3 g/kg/min
    Nitroglycerin IV infusion 1-2 min 3-5 min 5-100 g/min
    Diazoxide IV bolus or infusion 1-5 min 6-12 hr 50 mg IV every 5-10 min over 30 s, or 15-30 mg/min by IV infusion
    Hydralazine IV 10-20 min 3-8 hr 10-20 mg IV
      IM (also oral) 30 min 3-8 hr 10-50 mg IM
    Enalaprilat IV bolus 15-30 min 6 hr 1.25-5 mg
    Adrenergic Inhibitors        
    Labetalol IV 5 min 3-6 hr 0.5-2 mg/min IV infusion or 20-80 mg every 10 min to a maximum cumulative dose of 300 mg
    Trimethaphan IV infusion 1-5 min 10 min 0.5-5 mg/min
    Phentolamine IV bolus 1-2 min 3-10 min Load 5-15 mg IV every 5 min
    Esmolol IV bolus then infusion 1-2 min 10-20 min 250-500 g/kg bolus, then 50-300 g/kg/min infusion
    IV, intravenous; IM, intramuscular.
    From Chobanian AV, Bakris GL, Black GL, et al. The seventh report of the Joint National Committee on prevention, detection, evaluation,
    and treatment of high blood pressure. Hypertension 2003;42:1206-1252.

    Over the subsequent 3 months, blood pressure should be lowered gradually to less than 140/90 mm Hg (or to 130/80 mm Hg if there is diabetes or chronic kidney disease, as in your patient) with oral agents.

Suggested Readings

Appel LJ, Brands MW, Daniels SR, etal. Dietary approaches to prevent and treat hypertension. Hypertension 2006;47:296 308.

Bender KR, Filippone JD, Heitz S, etal. A systematic approach to hypertensive urgencies and emergencies. Curr Hypertens Rev 2005;1:275 281.

Bolli P, Myers M, McKay D. Canadian hypertension education program. Applying the 2005 Canadian hypertension education program recommendations: 1. Diagnosis of hypertension. CMAJ 2005;173:480 483.

Chobanian AV, Bakris GL, Black HR, etal. The seventh report of the Joint National Committee on prevention, detection, evaluation, and treatment of high blood pressure. JAMA 2003;289:2560 2572.

P.46


Hemmelgarn BR, Grover S, Feldman RD. Canadian hypertension education program. Applying the 2005 Canadian hypertension education program recommendations: 2. assessing and reducing global atherosclerotic risk among hypertensive patients. CMAJ 2005;173:593 595.

Khan NA, Hamet P, Lewanczuk RZ. Canadian hypertension education program. Applying the 2005 Canadian hypertension education program recommendations: 4. Managing uncomplicated hypertension. CMAJ 2005;173:865 867.

MacMahon S, Peto R, Cutler J, etal. Blood pressure, stroke, and coronary heart disease: part 1. prolonged differences in blood pressure: prospective observational studies corrected for the regression dilution bias. Lancet 1990;335:827 838.

Padwal R, Campbell N, Touyz RM. Canadian hypertension education program. Applying the 2005 Canadian hypertension education program recommendations: 3. Lifestyle modifications to prevent and treat hypertension. CMAJ 2005;173:749 751.

Tobe S, McAlister FA, Leiter L. Applying the 2005 Canadian hypertension education program recommendations: 5. Therapy for patients with hypertension and diabetes mellitus. CMAJ 2005;173:1154 1157.

Vasan RS, Beiser A, Sershadri S, etal. Residual lifetime risk for developing hypertension in middle-aged women and men: the Framingham heart study. JAMA 2002;287:1003 1010.

Wilson PW. Established risk factors and coronary artery disease: The Framingham Study. Am J Hypertens 1994;7:7S 12S.

ST-Elevation Myocardial Infarction

  • What are the major known risk factors for CAD?

  • What is the lifetime risk of ischemic heart disease deaths in men and women in the U.S. population?

  • What is the most common cause of acute MI?

  • In placebo-controlled trials, what types of treatments have been shown to improve outcome in patients with acute MI?

Discussion

  • What are the major known risk factors for CAD?

    The established major risk factors for CAD include smoking, hypertension, dyslipidemia specifically increased LDL cholesterol and low HDL cholesterol, diabetes mellitus, family history of CAD in a first-degree relative, male gender, and age. The first four factors are modifiable while the last three are not. There are other established risk factors that can be modified such as obesity, physical inactivity, an atherogenic diet, mental stress, and depression. The metabolic syndrome is also considered an important risk factor by the National Cholesterol Education Program (NCEP) guidelines. Emerging risk factors include high sensitivity C-reactive protein (hs-CRP), homocysteine, lipoprotein a, small dense LDL, prothrombotic factors fibrinogen, imbalance between tissue plasminogen activator and plasminogen activator inhibitor 1 (PAI-1), proinflammatory factors other than hs-CRP, and increased oxidative stress.

  • P.47


  • What is the lifetime risk of ischemic heart disease deaths in men and women in the U.S. population?

    The lifetime risk of developing CAD after age 40 is 49% in men and 32% in women. CAD is the leading cause of death in both men and women, accounting for approximately 20% of all deaths in the United States. However, women develop symptomatic CAD 10 to 15 years later than men.

  • What is the most common cause of acute MI?

    Plaque rupture, in the setting of coronary atherosclerosis, is the underlying cause of MI in most patients. At the site of plaque rupture, there is, in most cases, formation of an acute thrombus. Infrequent causes of MI include inflammation (arteritis), trauma, or coronary embolism.

  • In placebo-controlled trials, what types of treatments have been shown to improve outcome in patients with acute MI?

    Aspirin, clopidogrel, revascularization with thrombolytic agents or PCI, -adrenergic blockers, ACE inhibitors or ARBs, and aldosterone inhibitors (eplerenone) have all been shown to reduce mortality after MI. There are less compelling data regarding the utility of GIK (glucose, insulin, and potassium) solution infusion, and heparin and nitrate use. In the long term, lowering the LDL cholesterol with HMG-CoA reductase inhibitors (statins) and the use of warfarin have been shown to be beneficial. The benefit of eplerenone has been shown only in high-risk patients. The use of warfarin is cumbersome and not widespread. The use of aspirin imparts nearly the same survival benefit as the use of thrombolytics and other more expensive therapies. Finally, early treatment also imparts the highest benefit (the golden hour).

Case 1

A 62-year-old man with a history of hypertension is mowing his lawn at 9:00 a.m. on a Saturday morning when he experiences a heavy sensation in his chest. He stops mowing the lawn and within 10 minutes his symptoms resolve, and he resumes cutting the grass. Approximately 10 minutes later, he experiences severe, crushing chest pain associated with shortness of breath and pain radiating down his left arm. As he walks to his house, he becomes diaphoretic and nauseated, and vomits twice. At this point, he calls an ambulance and is taken to the ER. When you arrive to examine him, he is still experiencing severe pain. A 12-lead ECG reveals 3-mm ST-segment elevation in leads V2, V3, V4, and V5 with inferior ST-segment depression. The pain has been present for a total of approximately 45 minutes.

  • What initial actions should be taken in this patient?

  • Is this patient's hypertension a contraindication to thrombolytic therapy?

  • What are the risks associated with thrombolytic therapy and how long after the onset of acute MI is therapy beneficial?

  • Which is the better reperfusion therapy for acute MI thrombolytic therapy or primary percutaneous transluminal coronary angioplasty (PTCA)?

  • What therapies should be administered acutely with thrombolysis or primary PTCA?

  • P.48


  • What measures should be carried out before this patient is discharged?

  • Under what circumstances should the patients undergo coronary angiography if they did not undergo acute angioplasty and/or stenting on admission?

Case Discussion

  • What initial actions should be taken in this patient?

    The first actions that should be taken in this patient are to administer sublingual NTG, administer oxygen if oxygen saturation is below 90%, and establish venous access. IV -blockers and aspirin should be given. Analgesics such as morphine should be given if the pain does not resolve with NTG. Immediate transfer to the cardiac catheterization laboratory for coronary angioplasty and reperfusion of the infarct-related artery is the treatment of choice if it can be accomplished within 3 hours of the onset of chest pain. If not, thrombolytic agents should be administered immediately if there are no contraindications. Patients should be questioned about contraindications to thrombolytic agents before administration. Invasive procedures such as arterial puncture should be minimized if thrombolytic agents are to be administered to avoid bleeding. If the patient presents more than 3 hours following the onset of chest pain PCI is clearly preferable, because of the difficulty in lysing the clot after 3 hours. However, studies have shown that either thrombolytics or PCI is beneficial for at least 12 hours after the onset of pain.

  • Is this patient's hypertension a contraindication to thrombolytic therapy?

    Hypertension alone is not a contraindication to thrombolytic therapy. If the hypertension is uncontrolled and cannot be lowered quickly to a level below 180/110 mm Hg, the risk of intracranial bleeding is increased. These thrombolytic agents can still be considered in individual patients. Absolute contraindications to thrombolytic therapy include any prior intracranial hemorrhage (ICH), known cerebrovascular or intracranial neoplastic lesion, ischemic stroke within 3 months, active bleeding excluding menses, suspected aortic dissection, and significant closed head or facial trauma within 3 months. Relative contraindications in addition to uncontrolled hypertension are an old history of stroke, prolonged cardiopulmonary resuscitation (CPR) (more than 10 minutes) or major surgery within 3 weeks, internal bleeding in the last 4 weeks, active peptic ulcer, a known bleeding diathesis or use of anticoagulants, and pregnancy.

  • What are the risks associated with thrombolytic therapy and how long after the onset of acute MI is therapy beneficial?

    The major risk of thrombolytic therapy is bleeding. This risk is lowest with streptokinase, and highest with newer agents and when heparin is added to therapy. With alteplase-like agents, major bleeding occurs in approximately 5% of patients and ICH occurs in 0.9%. Factors that increase ICH include age (especially greater than 75 years), weight less than 70 kg, and hypertension (160/95 or higher) at presentation and the use of alteplase. Patients with more than three risk factors have two or three times higher risk of ICH. Patients with acute MI benefit from

    P.49


    thrombolytics for up to 12 hours after the onset of the infarction with earlier treatment leading to higher survival.

  • Which is the better reperfusion therapy for acute MI thrombolytic therapy or primary PTCA?

    PTCA is generally preferred over thrombolytic agents as a reperfusion therapy. Thrombolytics can be used in patients presenting early (<3 hours) especially when a catheterization laboratory is not readily available. Primary PTCA is preferred in most instances where there is rapid access to a skilled laboratory, especially in higher risk patients either due to cardiogenic shock or significant HF. It is also preferred in patients presenting later than 3 hours from symptom onset, when there are significant contraindications to thrombolytics or when the diagnosis is in doubt.

  • What therapies should be administered acutely with thrombolysis or primary PTCA?

    Chewable aspirin (162.5 mg) should be administered immediately once the diagnosis is made in all patients unless there is a contraindication to aspirin (i.e., aspirin allergy or active bleeding). IV -blockade should be instituted unless there are contraindications to their use such as pulmonary edema, significant atrioventricular block, heart rate less than 60 per minute, systolic blood pressure less than 100 mm Hg or significant bronchospasm and history of asthma. ACE inhibitors (or ARBs for allergic patients) should be begun within the first 24 hours in the absence of contraindications such as systolic blood pressure less than 100 mm Hg, renal insufficiency (serum creatinine greater than 3.0 mg/dL), or hyperkalemia. Clopidogrel should also be used during the hospital stay.

  • What measures should be carried out before this patient is discharged?

    Therapy with a statin should be started. LDL cholesterol levels fall after the first 24 hours after an acute MI, so lipid level measurements should be done within 24 hours of admission. Risk stratification with submaximal exercise test and assessment of left ventricular EF should be performed in patients who were not stratified by angiography. If a stent was placed during PTCA, clopidogrel is added for 3 to 6 months and perhaps longer if a drug-eluting stent was placed. Finally, an aldosterone antagonist should be added for patients with abnormal cardiac function and HF or diabetes.

    All patients should be counseled on smoking cessation and a low-fat diet. Each patient should be taught how to use NTG and should be instructed when to call for problems.

  • Under what circumstances should patients undergo coronary angiography if they did not undergo acute angioplasty and/or stenting on admission?

    Residual ischemic myocardium and low EF are major risk factors for mortality. This is why patients with recurrent ischemic chest pain, a positive submaximal exercise test, or an EF less than 40% usually undergo coronary angiography to determine if residual lesions causing ischemia can be corrected.

Case 2

A 67-year-old woman is in town visiting her children when she presents to your office complaining of severe symptoms of shortness of breath that has worsened over the last 12 hours. She tells you that she has had diabetes mellitus for the last 20 years and

P.50


hypertension that has been fairly well controlled for 15 years. Your examination reveals an S3 gallop and rales to her midscapular area. She also tells you that she has experienced recurrent chest heaviness over the last 2 days. When the ECG is done, there are Q waves in leads V2, V3, V4, and V5. A call to her regular physician reveals she had a normal ECG when he saw her 1 month ago.

  • At this point, what should you do?

  • What therapeutic interventions should be instituted at the time of admission?

  • Before discharge, she has an echocardiogram performed. What findings would favor long-term anticoagulant therapy with sodium warfarin?

  • Should this patient undergo coronary angiography or should she have a submaximal exercise test?

  • Would you recommend PTCA, surgery, or medical therapy

Case Discussion

  • At this point, what should you do?

    Your patient has had a recent anterior MI with left ventricular failure causing her symptoms. She needs to be hospitalized immediately, treated for HF, monitored for arrhythmias and recurrent ischemia, and risk-stratified. Thrombolytic therapy or PCI is not indicated because this is a completed infarction, nearly 48 hours old.

  • What therapeutic interventions should be instituted at the time of admission?

    Initial treatment consists of oxygen administration for hypoxemia and diuresis while avoiding hypokalemia. The goal of diuresis is to resolve pulmonary congestion. Aspirin should be started. Telemetry monitoring is necessary to detect arrhythmias. ACE inhibitors (or an ARB if allergic) should be started if the patient is not hypotensive and has no contraindications to their use. Aldosterone blockers should be introduced. Heparin should be considered in this patient with a large anterior MI because of the risk of left ventricular apical thrombus formation and embolism. A -blocker should be considered only after resolution of the patient's symptoms of HF.

  • Before discharge, she has an echocardiogram performed. What findings would favor long-term anticoagulant therapy with sodium warfarin?

    An apical thrombus, especially if mobile, increases the risk of embolism and is considered an acceptable indication for anticoagulation. The same is true of a dyskinetic or akinetic ventricular segment. In these cases, warfarin is continued for 3 to 6 months or until a thrombus is no longer present. However, these recommendations are not based on prospective randomized trials.

    Two clear indications for anticoagulation in this setting are the presence of atrial fibrillation or a history of a previous embolic episode.

  • Should this patient undergo coronary angiography or should she have a submaximal exercise test?

    This woman presented with a large MI and HF suggesting severe CAD. Her mortality risk is high and therefore exercise testing for risk stratification is not necessary. She should therefore be evaluated directly with coronary angiography.

    P.51


    Coronary angiography shows a 90% proximal right coronary artery obstruction, a 90% proximal left anterior descending (LAD) obstruction, and a 100% proximal circumflex obstruction. Her EF by left ventricular angiography is 34%, with moderate anterior hypokinesis.

  • Would you recommend PTCA, surgery, or medical therapy?

    With severe three-vessel disease and left ventricular dysfunction, coronary artery bypass surgery is indicated in this patient. The presence of diabetes favors surgery over PTCA in this particular case even if the EF is not low.

Suggested Readings

Antman EM, Anbe DT, Armstrong PW, etal. ACC/AHA guidelines for the management of patients with ST-elevation myocardial infarction executive summary: a report of the American College of Cardiology/American Heart Association Task Force on Practice Guidelines (Writing Committee to Revise the 1999 guidelines for the management of patients with acute myocardial infarction). Circulation 2004;110:588 636.

Keeley EC, Boura JA, Grines CL. Primary angioplasty versus intravenous thrombolytic therapy for acute myocardial infarction: a quantitative review of 23 randomized trials. Lancet 2003;361:13 20.

Libby P. Current concepts of the pathogenesis of the acute coronary syndromes. Circulation 2001;104:365 372.

Verma VK, Hollenberg SM. Update on acute coronary syndromes and ST-elevation myocardial infarction. Curr Opin Crit Care 2005;11:401 405.

Unstable Angina and Non ST-Elevation Myocardial Infarction

  • What is an ACS?

  • What is unstable angina?

  • What is a non ST-elevation myocardial infarction (NSTEMI)?

  • What common pathophysiologic processes underlie both unstable angina and NSTEMI?

  • What is the estimated incidence of silent ischemic episodes in the setting of unstable angina?

  • What measures have been shown to improve the clinical outcome in the setting of ACSs?

Discussion

  • What is an ACS?

    The ACS spectrum includes unstable angina, NSTEMI, and ST-elevation myocardial infarction (STEMI). Patients having ACS present to the ER with chest pain. The pathophysiology of the syndrome is similar in these patients.

  • P.52


  • What is unstable angina?

    Stable angina is a stable pattern of chest or arm discomfort caused by similar degrees of physical or emotional stress. Unstable angina is angina that occurs at rest or with minimal exertion, or is of recent onset (less than 1 month) or has a crescendo quality (i.e., occurs more frequently), and is more severe or of longer duration.

  • What is an NSTEMI?

    This definition evolved from the old description of non Q-wave MI. It applies to patients who have a presentation similar to unstable angina, especially with prolonged pain at rest, and who have evidence of myocardial necrosis. They are distinct from those with STEMI because they do not have persistent ST elevation on presentation. Instead, either ST depression or T-wave changes is more common. This classification has therapeutic implications because Q waves do not always indicate a transmural MI and subendocardial necrosis occurs only in 50% of the cases of non Q-wave MI. Therefore, the new classification of STEMI and NSTEMI was introduced. NSTEMI is frequently seen in the elderly, those with a previous MI, and compared with STEMI is more commonly associated with incomplete occlusion of the coronary artery. This definition allows patients with STEMI to go urgently for revascularization while NSTEMI patients can often wait. Early mortality is higher with STEMI. A Q-wave or a non Q-wave MI can be caused by either STEMI or NSTEMI. Patients with STEMI have higher in-hospital mortality whereas those with NSTEMI have higher reinfarction and mortality rates in the subsequent 6 months to 1 year.

  • What common pathophysiologic processes underlie both unstable angina and NSTEMI?

    By far the most common mechanism of unstable angina/NSTEMI is atherosclerosis-related coronary plaque rupture, usually with superimposed thrombus. Other possible mechanisms include coronary spasm or inflammation as well as increased myocardial oxygen requirements. These vulnerable plaques are lipid rich with a thin fibrous cap. Infiltration of this cap with inflammatory cells leads to its disruption, followed by exposure of the subendothelial matrix to the blood stream with platelet activation and aggregation. This leads initially to the formation of a platelet-rich (gray) thrombus. Later, however, there is also activation of the coagulation cascade with formation of a fibrin (red) thrombus. In NSTEMI, this thrombus is nonocclusive in most cases.

  • What is the estimated incidence of silent ischemic episodes in the setting of unstable angina?

    Silent ischemia is the presence of ischemic ECG changes without angina. The reported incidence after unstable angina/NSTEMI varies from one fourth to two thirds of the patients. It is detected by continuous ECG monitoring using a holter monitor. Silent ischemia is frequently preceded by increases in blood pressure and heart rate, causing increased myocardial oxygen consumption. It is associated with higher death and MI rates and should be treated once detected.

  • P.53


  • What measures have been shown to improve the clinical outcome in the setting of ACSs?

    During the acute phase of care, the use of aspirin, thienopyridine derivatives (e.g., clopidogrel), glycoprotein IIb/IIIa inhibitors, unfractionated and low-molecular-weight heparins, and direct thrombin inhibitors (e.g., lepirudin) as well as revascularization (especially in high-risk patients) have been shown to reduce the rate of MI and death. The use of -blockers was shown to reduce recurrent infarctions. The use of certain calcium channel blockers in patients intolerant to -blockers has also been shown to reduce recurrent MI. In the intermediate and long terms, the use of aspirin, clopidogrel, statins, ACE inhibitors, and -blockers is recommended to reduce mortality and recurrent MI. The concomitant use of warfarin has been shown to improve outcomes but its use is limited to patients who have another indication for warfarin (such as atrial fibrillation). Modification of coronary risk factors is also warranted.

Case 1

A 42-year-old registered nurse is seen because of pain in the chest. She describes a pain in my heart and points to a 1-cm2 area above the left breast. The pain is intensified by deep breathing, coughing, recumbency, and twisting motions. It has lasted continuously for 2 days. Three days ago, she noted extreme fatigue and shortness of breath lasting for 24 hours. Findings from a complete physical examination are normal.

  • What is the most likely diagnosis in this patient, and why?

    As you are about to discharge this patient, her husband tells you he is concerned about his wife because her sister underwent coronary bypass surgery at 44 years and her brother at 34 years. Because the pain has some features of pericarditis, you decide to do an ECG. It shows normal sinus rhythm with Q waves in the inferior leads and diffuse ST-segment elevation.

  • What is your diagnosis, and what would you do?

Case Discussion

  • What is the most likely diagnosis in this patient, and why?

    Chest wall pain or pericarditis would be the most likely initial diagnosis in this patient. Angina pectoris is uncommon in women in this age-group, and this pain is not anginal in character. Aortic dissection pain is typically very severe from the start, sharp, tearing and can radiate to the back. Acute cholecystitis manifests clinically with right upper quadrant tenderness and occasionally a palpable gallbladder. Pneumonia and pleurisy are differentiated because of the association with fever and cough with abnormal chest examination. A pneumothorax is associated with acute shortness of breath findings of hyperresonance to percussion and diminished breath sounds on the affected side. Pain arising from the chest wall is the most common cause of chest pain in any age-group, and often has no discernible cause. It can be reproduced by pressure over the painful area. Pericarditis is often accompanied

    P.54


    by a friction rub. This rub has a coarse, leathery, or walking on crunchy snow sound, with accentuation during systole as well as early and late diastole (however, sometimes only one or two components are audible). Inspiration intensifies the rub. Its features often include a precise localization and tenderness on palpation over the affected area. Deep breathing, position changes, and specific body movements such as twisting often accentuate the pain. Its duration varies from a few seconds to days. Therapy is nonspecific, consisting of reassurance and simple analgesics or nonsteroidal drugs.

  • What is your diagnosis, and what do you do?

    This patient probably had a silent inferior MI a few days ago and now presents with postinfarction pericarditis. The Q waves are inferiorly related to the MI while the diffuse ST-segment elevation is compatible with pericarditis. Additional history is that she has had type 2 diabetes mellitus for 20 years and a recent cholesterol screening at a health fair. Her LDL cholesterol was 242 mg/dL, consistent with familial heterozygous hypercholesteremia.

    The patient should be admitted for telemetry observation. Troponin I is likely to be elevated and should be drawn. When you ask the patient to sit up, lean forward, and exhale, a two-component pericardial friction rub is noted. Aspirin should be given for her MI and can be used at much higher doses to treat the concomitant pericarditis. Ibuprofen is the NSAID of choice for pericarditis; however, its use should be avoided in the setting of an acute MI as it could interfere with scar formation. The remainder of her treatment is as discussed for patients with acute MI, except that this infarction is older and acute reperfusion is not indicated. Silent ischemia is more common in diabetes. Although women younger than 50 years do not often have symptomatic coronary disease, this advantage is neutralized by the presence of diabetes. When evaluating patients with chest pain, attention to CAD risk factors is paramount.

Case 2

A 57-year-old automobile salesman who is hypertensive and a heavy cigarette smoker describes a pressure-like sensation that developed for the first time 3 weeks before. The discomfort, which begins in the retrosternal area, radiates to the left side of his lower jaw, occurs when he walks rapidly in cold air, and more recently occurs at rest. Careful history reveals that it lasts for 10 to 15 minutes, but an especially severe episode awakened him the night before and lasted nearly half an hour before resolving spontaneously. Except for a blood pressure of 150/100 mm Hg, the physical examination findings are normal. An ECG (obtained after the pain has disappeared) reveals deep and symmetric T-wave inversion in leads V1 to V4. The patient is admitted and given IV heparin and oral aspirin.

  • What is your diagnosis?

  • What are some common physical findings during an ischemic episode?

    Approximately 4 hours after admission, the patient again experiences transient chest pressure. You order an ECG. The T waves are now upright in leads V1 to V4.

  • What are these ECG changes called, and what do they represent?

  • How should the recurrent chest pain be treated?

  • What should be done next?

P.55


Case Discussion

  • What is your diagnosis?

    This patient has either unstable angina or an NSTEMI. The pain is both new in onset and occurs at rest. The T-wave inversions confirm the diagnosis of ischemia. The results of cardiac enzyme tests separate unstable angina (negative enzymes) from NSTEMI (positive enzymes).

  • What are some common physical findings during an ischemic attack?

    Increases in heart rate and blood pressure are the most common findings during ischemia. Physical examination performed during an ischemic attack may reveal an S4 or a murmur of mitral regurgitation. If the ischemic area is large then an S3, pulmonary rales, a dyskinetic apical impulse, and hypotension could be noted. Ischemia decreases left ventricular compliance (increased stiffness ) with subsequent increase in left ventricular filling pressure. The resistance to filling during atrial contraction is what produces the S4 sound. However, an S4 is a nonspecific finding and is frequently heard in older adults. Localized contraction abnormalities may produce transient papillary muscle dysfunction and failure of complete apposition of the leaflets, resulting in mitral regurgitation. Similar contraction abnormalities can cause an outward bulge of the left ventricle with dyskinetic apical impulse. This can be felt by using the palm of the hand while the patient is in the left lateral decubitus position.

  • What are these ECG changes called, and what do they represent?

    When previously inverted T waves become upright in the presence of chest pain, it is called pseudonormalization. This is strongly suggestive of ischemia.

  • How should the recurrent chest pain be treated?

    The pseudonormalization of the T waves clearly indicates myocardial ischemia. This pain should be treated with sublingual NTG and IV morphine, followed by IV NTG and -blockade if there are no contraindications. The patient is already receiving aspirin and heparin (low-molecular-weight heparin can also be used in this case). Platelet glycoprotein IIb/IIIa inhibitors are of value in the treatment of high-risk patients with unstable angina/NSTEMI. Statins have been shown to reduce cardiac event rates when used acutely in this population. If the patient is not a candidate for coronary artery bypass graft (CABG), then clopidogrel should be considered.

  • What should be done next?

    This patient had signs of myocardial ischemia on admission with recurrent pain on IV heparin. This is suspicious for the presence of substantial ischemia, and the deeply inverted T waves in leads V1 to V4 mostly likely represent a high-grade proximal LAD stenosis. This patient should undergo coronary angiography. PCI should be performed in general for one- or two-vessel disease and normal or near-normal left ventricular function. For most patients with three-vessel disease or left ventricular dysfunction, especially in the presence of diabetes, coronary artery bypass surgery (using whenever possible an internal mammary artery graft to the LAD) is indicated. The use of PCI versus coronary bypass surgery may vary depending on patient or physician preference, lesion anatomy, the presence of proximal LAD disease, or the patient's comorbidities.

P.56


Suggested Readings

Antmann EM, Cohen M, Bernink PJLM, etal. The TIMI risk score for unstable angina/non-ST elevation MI. A method for prognostication and therapeutic decision making. JAMA 2000;284:835 842.

Boden WE, McKay RG. Optimal treatment of acute coronary syndromes- an evolving strategy. N Engl J Med 2001;344:1939 1942.

Braunwald E, Antman EM, Beasley JW, etal. American College of Cardiology/American Heart Association Task Force on Practice Guidelines (Committee on the Management of Patients With Unstable Angina). ACC/AHA guideline update for the management of patients with unstable angina and non-ST-segment elevation myocardial infarction 2002: summary article: a report of the American College of Cardiology/American Heart Association Task Force on Practice Guidelines (Committee on the Management of Patients With Unstable Angina). Circulation 2002;106:1893 1900.

Cannon CP, Weintraub WS, Demopoulos LA, etal. Comparison of early invasive and conservative strategies in patients with unstable coronary syndromes treated with the glycoprotein IIb/IIIa inhibitor tirofiban. For the TACTICS-thrombolysis in myocardial infarction 18 investigators. N Engl J Med 2001;344:1879 1887.

Cohn PF, Fox KM, Daly C. Silent myocardial ischemia. Circulation 2003;108:1263.

Fuster V, Moreno PR, Fayad ZA, etal. Atherothrombosis and the high-risk plaque. Part I: evolving concepts. J Am Coll Cardiol 2005;46:937 954.

Sudden Cardiac Death

  • What kind of heart disease is seen most commonly in adults who die suddenly? In young athletes?

  • Which types of arrhythmias are associated with cardiac arrest and sudden cardiac death (SCD)?

  • Which patients are at highest risk for SCD?

  • What is the cause of SCD in the long QT syndrome?

Discussion

  • What kind of heart disease is seen most commonly in adults who die suddenly? In young athletes?

    Approximately 90% of cases of SCD are due to ventricular fibrillation in the setting of preexisting structural heart disease; 5% to 10% occur in the absence of organic heart disease. In individuals younger than 30 years and young athletes, SCD is very rare, but when it does occur it is usually due to hypertrophic cardiomyopathy. Arrhythmogenic right ventricular dysplasia and acute myocarditis are other infrequent causes of SCD in the young adult. After the age of 40, 65% to 70% of all SCDs are attributable to CAD.

  • P.57


  • Which types of arrhythmias are associated with cardiac arrest and SCD?

    In the field, paramedics most commonly record ventricular fibrillation or ventricular tachycardia during cardiac arrest. Less frequently seen, and associated with a poorer prognosis, are bradyarrhythmias, asystole, and pulseless electrical activity (electrical mechanical dissociation).

    Cardiac arrest due to any arrhythmia results in rapid depletion of oxygen in vital organs. After 6 minutes, brain damage is expected to occur, except in cases of hypothermia. Therefore, early CPR and rapid advanced cardiac life support (ACLS), with defibrillation, are essential in improving survival and neurologic recovery.

  • Which patients are at highest risk for SCD?

    Patients at highest risk for SCD are those who have previously survived an episode of SCD or have a history of rapid, sustained ventricular tachycardia especially in the setting of reduced left ventricular function. One of the best measures of risk of SCD is left ventricular function and the risk of SCD increases as left ventricular function decreases.

    The incidence of SCD is greater in men than in women. This increased risk remains despite adjusting for the presence of comorbidities such as ischemic heart disease and age. The risk of SCD increases with age.

  • What is the cause of SCD in the long QT syndrome?

    The long QT syndrome is a cause of syncope and SCD in patients with structurally normal hearts, but may also result in SCD in patients with structural heart disease. The long QT syndrome can be either acquired or inherited. Several genetic defects involving cardiac ion channels have been identified in families with inherited long QT syndrome. In acquired long QT syndrome, several classes of drugs that affect cardiac ion channels and several medical conditions associated with electrolyte abnormalities have been identified. In both inherited and acquired forms, cardiac repolarization is prolonged, and reflected in a long QT interval on the ECG. Syncope and SCD in long QT syndrome are caused by a specific, polymorphic ventricular tachycardia called torsade de pointes ( twisting of the points ). Drugs that prolong the QT interval (e.g., phenothiazines, tricyclic antidepressants, and certain antiarrhythmics) are particularly likely to cause SCD in patients with another cause of prolonged QT interval or in patients with structural heart disease.

Case

A 65-year-old man complains of chest discomfort on the golf course and within seconds collapses and is unresponsive. His companions initiate bystander CPR and an ambulance is called. Paramedics arrive within 10 minutes. A quick look at the rhythm using the defibrillator paddles reveals ventricular fibrillation. After one shock at 200 J using a biphasic fibrillator, sinus rhythm is restored and a pulse is felt. The patient is transported to the hospital. Initial ECG shows Q waves in the precordial leads, and diffuse, nonspecific ST-segment and T-wave abnormalities, with a normal QT interval. Serum electrolytes are

P.58


normal. Initial and subsequent cardiac enzyme determinations do not indicate evidence of an acute MI. Family members state that the patient was not on cardiac medications and had no cardiac history. The patient, initially unresponsive and requiring mechanical ventilation, recovers neurologically over the next 48 hours and is extubated. Apart from a mild short-term memory deficit, he seems to be back to his usual self and none the worse for the experience. Troponin was not elevated during the hospitalization.

  • What tests should be performed on this patient now?

  • Will coronary revascularization be of benefit in preventing recurrent SCD in this patient?

  • Is there a role for electrophysiologic testing in this patient?

  • What is the best treatment to prevent recurrent SCD in this patient?

Case Discussion

  • What tests should be performed on this patient now?

    Despite the near-miraculous recovery of this patient, his risk of recurrent SCD is high and measures should be taken to identify the cause of SCD in his case and prevent recurrence. Although most cases of SCD in his age-group are related to CAD, it can be unclear in a particular patient whether the initiating event was a primary arrhythmia or ischemia. Ischemia can be due to increased metabolic demands (e.g., exercise) in the face of a fixed coronary obstruction, or due to transient decreased coronary blood flow caused by atherosclerotic plaque rupture or coronary vasospasm. It is likely that many episodes of SCD are multifactorial, superimposing transient triggering events (e.g., ischemia, changes in autonomic tone, electrolyte abnormalities, or premature ventricular complexes) on an arrhythmogenic substrate such as the cell damage created by a previous MI. This patient's complaint of chest discomfort before collapse may on first consideration suggest ischemia as the initiating event, but patients with coronary disease who have ventricular tachycardia sometimes complain of chest pain because they become ischemic secondary to the rapid heart rate. The ECG evidence of an anterior infarction without the enzyme changes characteristic of acute infarction suggests that, despite his negative cardiac history, he may have had a previous silent MI. This old ventricular scar may be a substrate for a primary reentrant ventricular tachycardia. To define his cardiac disease better, including his left ventricular function, and to determine if he has a substrate for recurrent ischemia, cardiac catheterization should be performed. In a French study of 84 survivors of out-of-hospital cardiac arrest without obvious noncardiac cause of the arrest, immediate coronary angiography with angioplasty was shown to be safe with potential long-term benefit when performed by an experienced team. Cardiac catheterization in this patient showed a 100% proximal LAD artery occlusion as well as a 90% occlusion of the first obtuse marginal branch of the left circumflex artery. The right coronary artery was normal. The left ventricular EF was reduced at 30% (normal 55%). There was an anteroapical left ventricular aneurysm. The lateral wall of the left ventricle (supplied by the left circumflex artery) had normal motion.

  • P.59


  • Will coronary revascularization be of benefit in preventing recurrent SCD in this patient?

    As discussed previously, SCD may be multifactorial and it is difficult to determine the precise triggers for an SCD episode. This patient has evidence on catheterization of a previous anterior infarction, with an anteroapical left ventricular aneurysm. This aneurysm may be a substrate for reentrant-sustained monomorphic ventricular tachycardia. In addition, he has a significant stenosis in an obtuse marginal artery, with normal left ventricular wall motion in the region served by this artery. This is a substrate for ischemia. In an attempt to correct possible triggering factors like ischemia, it would be reasonable to dilate the obtuse marginal stenosis with balloon angioplasty, and this was performed in this patient. Unfortunately, in patients with reduced left ventricular function who have an aborted episode of SCD, there is no evidence that antiischemia measures alone prevent recurrent SCD.

  • Is there a role for electrophysiologic testing in this patient?

    Most patients with similar presentations have inducible sustained monomorphic ventricular tachycardia during electrophysiologic testing. In the past, many such patients underwent electrophysiologic testing and were then treated with antiarrhythmic drug therapy guided by serial electrophysiologic testing. Patients who failed drug therapy or who did not have inducible sustained monomorphic ventricular tachycardia were treated with empiric amiodarone or an implantable defibrillator. A number of randomized controlled clinical trials have been performed that compare the efficacy of the implantable defibrillator with antiarrhythmic drug therapy. All these trials suggest that prophylactic therapy with implantable defibrillator is superior to antiarrhythmic drug therapy guided by electrophysiologic testing or empiric amiodarone in preventing recurrent SCD. In the AVID (Antiarrhythmics Versus Implantable Defibrillators) trial, patients who were enrolled had hemodynamically significant sustained ventricular tachycardia and a left ventricular EF of 40% or less, or ventricular fibrillation. There was a 31% reduction in the total mortality rate after 3 years with implantable defibrillators compared with antiarrhythmic drug therapy. The trial was stopped early when a survival benefit was noted in patients receiving the ICD compared with those treated with amiodarone or sotalol. The MADIT II (Multicenter Automatic Defibrillator Implantation Trial) demonstrated that ICDs significantly improve survival in patients with CAD and left ventricular EF of 30% or less. In those patients who have survived SCD and/or have reduced LV function, an ICD is the treatment of choice. Electrophysiologic testing is usually not performed in SCD survivors, and was not performed in this patient.

  • What is the best treatment to prevent recurrent SCD in this patient?

    This patient was a good candidate for an implantable defibrillator, and he received one. The defibrillator was implanted in the left pectoral region in the electrophysiology laboratory, using local anesthesia and conscious sedation. The patient was discharged home the day after the implant. In addition, this patient received other medical therapy that has been shown to reduce the risk of SCD. -Blocking drugs have been shown to reduce total mortality after MI as well as improve pump function in some cases. Aspirin may help prevent reinfarction. ACE inhibitors have

    P.60


    been shown to improve survival in patients with reduced left ventricular function. Spironolactone, a mineralocorticoid receptor antagonist has been similarly shown to reduce all cause mortality and SCD in patients with left ventricular systolic function of 35% or less. Lipid-lowering agents may prevent progression of atherosclerosis and should be used in patients with lipid abnormalities. Other risk factor modifications such as smoking cessation would be recommended. Amiodarone may reduce SCD after MI, but it does not clearly reduce total mortality. In the SCD-HeFT study, amiodarone was shown to be not beneficial in preventing SCDs in patients with a left ventricular systolic function of 35% or less compared with the placebo group in both ischemic and nonischemic patients. In this patient, there would be no added value in using amiodarone because he already has an implantable defibrillator. In fact, amiodarone, by its effect of increasing the electrical defibrillation threshold, might actually interfere with the function of the defibrillator, and should therefore be avoided. However, because ICDs do not prevent arrhythmias, patients who have frequent device discharges from recurrent arrhythmias may benefit from adjunctive antiarrhythmic drug therapy, like amiodarone. Such treatment, by reducing the frequency of appropriate shocks, improves the patient's quality of life.

Suggested Readings

The Antiarrhythmics Versus Implantable Defibrillators (AVID) Investigators. A comparison of antiarrhythmic drug therapy with implantable defibrillators in patients resuscitated from near fatal ventricular arrhythmias. N Engl J Med 1997;337:1576.

Bardy GH, Lee KL, Mark DB, etal. Amiodarone or an implantable cardioverter-defibrillator for congestive heart failure. N Engl J Med 2005;352:225.

Huikuri HV, Castellanos A, Myerburg RJ. Sudden death due to cardiac arrhythmias. N Engl J Med 2001;345:1473 1482.

Maron BJ, Shirani J, Poliac LC, etal. Sudden death in young competitive athletes: clinical, demographic, and pathological profiles. JAMA 1996;276(3):199 204.

Moss AJ, Long QT. Syndrome. JAMA 2003;289:2041 2044.

Moss AJ, Zareba W, Hall WJ, etal. Prophylactic implantation of a defibrillator in patients with myocardial infarction and reduced ejection fraction. N Engl J Med 2002;346:877.

Spaulding CM, Joly LM, Rosenberg A, etal. Immediate coronary angiography in survivors of out-of-hospital cardiac arrest. N Engl J Med 1997;336:1629.

Zheng ZJ, Croft JB, Giles WH, etal. Sudden cardiac death in the United States, 1989 to 1998. Circulation 2001;104:2158.

Valvular Heart Disease

  • What is the difference between valvular insufficiency and valvular regurgitation?

  • What types of myocardial hypertrophy can result from valvular abnormalities?

  • What is the most serious long-term consequence of either concentric or eccentric hypertrophy?

  • P.61


  • What is the relationship between the pressure gradient across a stenotic valve, the blood flow across the valve, and the valve area?

Discussion

  • What is the difference between valvular insufficiency and valvular regurgitation?

    Regurgitation and insufficiency are interchangeable terms to describe backward flow of blood across a valve at a time in the cardiac cycle when there would be no significant flow across a competent valve.

  • What types of myocardial hypertrophy can result from valvular abnormalities?

    When there is a pressure load imposed on the ventricle (such as aortic stenosis for the left ventricle or pulmonic stenosis for the right ventricle), concentric hypertrophy develops. Concentric hypertrophy means that the myocardial wall thickness is increased with a normal or decreased internal ventricular diameter. A volume load (such as aortic insufficiency or mitral regurgitation for the left ventricle or tricuspid regurgitation for the right ventricle) results in eccentric hypertrophy; the wall thickness is normal but the internal diameter of the ventricle is increased. Overall, left ventricular mass is increased in both types of hypertrophy.

  • What is the most serious long-term consequence of either concentric or eccentric hypertrophy?

    With long-standing hypertrophy of either type, myocardial dysfunction may occur resulting in HF.

  • What is the relationship between the pressure gradient across a stenotic valve, the blood flow across the valve, and the valve area?

    The pressure gradient across a valve is proportional to the blood flow across the valve divided by the valve area. The pressure gradient is a result of the flow across the stenotic valve and the degree of stenosis. For example, if flow remains the same and the valve becomes more stenotic over time, the pressure gradient increases. However, an increased pressure gradient may not always mean that stenosis has progressed. For example, blood flow may increase with fever or anemia, resulting in an increased pressure gradient without any change in valve area. On the other hand, a decrease in the pressure gradient does not mean an improvement in valve stenosis. For example, if myocardial function deteriorates and blood flow (cardiac output) decreases, the pressure gradient decreases. This decrease, however, does not reflect a less stenotic valve, but indicates a deterioration in myocardial function because the heart can no longer pump the same blood flow.

Case 1

A previously healthy but inactive 42-year-old man is seen in the ER after a first episode of syncope, which occurred while he was playing basketball. On questioning, he describes a 2-month history of exertional chest pain. He has not seen a physician during his adult life. Physical examination reveals the following findings. His supine blood pressure is 116/80 mm Hg without any significant orthostatic change. There is no jugular venous

P.62


distention, but there are slowly rising, small-amplitude, and somewhat sustained carotid pulses. His lungs are clear. A sustained and slightly laterally displaced apex impulse is noted, as well as a soft first heart sound and a single second heart sound, a prominent fourth heart sound, and a grade 3/6 harsh, late-peaking, crescendo decrescendo systolic murmur heard best at the cardiac base and radiating to the carotids with a high-frequency component at the cardiac apex. No clubbing, cyanosis, or edema is noted.

  • What is the most likely valvular lesion in this patient?

  • What is the most likely cause of aortic stenosis in this age-group?

  • What is the average survival of patients with uncorrected aortic stenosis after the onset of syncope?

  • How is the severity of aortic stenosis most accurately determined?

  • What is the best therapy for symptomatic aortic stenosis?

Case Discussion

  • What is the most likely valvular lesion in this patient?

    The history of angina and syncope and the classic physical examination findings make aortic stenosis an almost certain diagnosis in this patient. The characteristic arterial pulses described have been referred to as pulsus parvus et tardus. The single second heart sound indicates the absence of the aortic component, suggesting severe immobility of the aortic valve. The murmur is also characteristic of aortic stenosis with its crescendo decrescendo quality and the late peaking. Do not be fooled by the high-frequency component at the cardiac apex. Although the murmur of aortic stenosis is most often heard at the upper cardiac border with radiation to the carotid arteries, the murmur may also radiate to the apex, where it may be mistaken for the murmur of mitral regurgitation.

  • What is the most likely cause of aortic stenosis in this age-group?

    Between 35 and 65 years of age, degenerative change in a congenitally bicuspid aortic valve is the predominant cause of aortic stenosis. Beyond 65 years of age, aortic stenosis usually results from calcification of a previously normal tricuspid aortic valve (senile calcific aortic stenosis). Although the exact cause of senile aortic stenosis is unknown, it is associated with hypertension and hyperlipidemia. Isolated aortic stenosis in the United States rarely results from rheumatic disease.

  • What is the average survival of patients with uncorrected aortic stenosis after the onset of syncope?

    Patients with aortic stenosis may remain asymptomatic for years, but once symptoms develop the course of the disease may be quite fulminant. According to studies conducted before valve surgery was available, such patients with syncope due to aortic stenosis could expect to survive an average of 3 years after the onset of syncope. The average survival after the onset of angina pectoris or HF is 5 and 2 years, respectively. Therefore, the onset of angina, syncope, or HF due to aortic stenosis signals the need for valve replacement. Patients should also be questioned

    P.63


    about more subtle symptoms such as exertional dyspnea or a decrease in exercise capacity, as these symptoms may also indicate the need for surgery.

  • How is the severity of aortic stenosis most accurately determined?

    The severity of aortic stenosis can be precisely determined by either cardiac catheterization or Doppler/echocardiography. Both techniques can provide accurate estimations of the pressure gradient and the aortic valve area. Doppler echocardiography is generally used for evaluation and follow-up because it is noninvasive and easily repeated. The normal aortic valve area is 3 cm2. Mild, moderate, and severe aortic stenosis are present when the valve area is >1.5 cm2, between 1.5 and 1 cm2, and <1 cm2, respectively. Aortic stenosis is said to be critical when the valve area is 0.7 cm2 or less. Pressure gradient measurements alone are not adequate to determine the severity of aortic stenosis because, as already discussed, pressure gradients are determined by both the area of the stenotic valve and the blood flow across the valve. Physical examination findings such as the late-peaking murmur and the absent aortic component of the second heart sound may be suggestive of severe aortic stenosis but are poorly sensitive and specific compared with the information yielded by the aortic valve area. Echocardiography can also evaluate ventricular function and hypertrophy as well as provide information about the etiology of the aortic stenosis.

  • What is the best therapy for symptomatic aortic stenosis?

    Aortic valve replacement is the best therapy for symptomatic aortic stenosis. In symptomatic patients with severe aortic stenosis, aortic valve replacement results in a postoperative survival that is close to that of the general population. Older patients also generally have a good survival following aortic valve replacement for aortic stenosis.

    Long-term results of balloon aortic valvuloplasty (a catheter-based procedure) have been disappointing. Therefore, it is used primarily for palliation in patients who are not candidates for aortic valve replacement because of other medical problems or as a bridge to aortic valve replacement in patients deemed too ill for surgery. However, serious complications and mortality are high in these patients and restenosis generally recurs within 6 to 12 months.

Case 2

A 50-year-old woman who had an innocent murmur diagnosed in childhood presents with dyspnea on exertion, orthopnea, and paroxysmal nocturnal dyspnea of several months' duration. On questioning, she describes a 1-year history of fatigue and exhaustion that has limited her daily activities. She has not seen a physician in years.

On physical examination, her blood pressure is 110/70 mm Hg. Her jugular venous pressure is normal and she has mildly diminished arterial pulse amplitude with a normal arterial upstroke. Her lungs are clear to percussion and auscultation. There is a laterally displaced apex impulse and a palpable third heart sound that is easily heard. The first heart sound is soft and there is a widely split second heart sound with normal respiratory splitting. A grade 3/4 blowing, high-pitched systolic murmur is heard at the apex and radiates to the axilla and left infrascapular area. There is trace edema but no clubbing or cyanosis.

P.64


  • What is the valvular lesion in this patient?

  • What is the most common underlying cause of severe mitral regurgitation in the adult U.S. population?

  • Does medical therapy prevent progression of mitral regurgitation?

  • When should surgery be considered for patients with severe mitral regurgitation?

  • What are the choices for mitral valve surgery?

Case Discussion

  • What is the valvular lesion in this patient?

    Chronic mitral regurgitation, the most insidious of all left-sided valvular lesions, is the most likely diagnosis in this patient. Severe left ventricular dysfunction is not uncommon at presentation in this disorder. The holosystolic apical murmur is characteristic of chronic mitral regurgitation. The third heart sound suggests that the mitral regurgitation is severe, and is a reflection of the large volume of blood crossing the mitral valve in early diastole. However, the third heart sound does not necessarily imply HF. In chronic mitral regurgitation, the large regurgitant volume entering the left atrium results in left atrial enlargement with the left atrium often touching the spine. The murmur is transmitted to the spine through the left atrium, accounting for its radiation to the subscapular area.

    The murmur described is typical for chronic mitral regurgitation, but must be distinguished from such murmurs as tricuspid regurgitation, aortic stenosis, and VSD. The murmur characteristic of aortic stenosis is distinguished by its quality (crescendo decrescendo), location, and radiation, as described in Case 1 under section on Valvular Heart Disease. A tricuspid insufficiency murmur is usually well localized to the left sternal border with little radiation, and has the characteristic feature of an increase in intensity with inspiration. The murmur characteristic of VSD is typically heard best at the left sternal border, often has a harsh quality, and does not change with respiration. The murmur of VSD is rarely heard in adults because most congenital VSDs are detected in childhood and resolve spontaneously or are surgically corrected. A VSD is a rare complication of acute MI in adults.

  • What is the most common underlying cause of severe mitral regurgitation in the adult U.S. population?

    Myxomatous mitral valve disease, usually as an isolated lesion or sometimes associated with other connective tissue disorders (e.g., Marfan's and Ehlers-Danlos syndromes), constitutes the most common cause of severe mitral regurgitation necessitating mitral valve replacement or repair in the United States, especially in younger people. A smaller number of cases are due to rheumatic heart disease, infective endocarditis, or spontaneously ruptured chordae tendineae. In older people, severe mitral regurgitation often accompanies left ventricular enlargement and dysfunction due to CAD and MI. The mitral regurgitation is not due to an abnormality

    P.65


    of the valve leaflets but due to a combination of abnormalities of the supporting structures of the valve including stretching of the mitral annulus along with papillary muscle dysfunction and wall motion abnormalities.

  • Does medical therapy prevent progression of mitral regurgitation?

    There are no clinical trials demonstrating benefits of any medical therapy for chronic mitral regurgitation that is due to a primary abnormality of the mitral valve. If mitral regurgitation is due to left ventricular enlargement with dysfunction of the mitral valve apparatus, ACE inhibitors and -blockers are indicated to improve ventricular function. If ventricular function improves in patients with mitral regurgitation due to ventricular dysfunction, the mitral regurgitation will improve as the valve apparatus becomes more functional.

  • When should surgery be considered for patients with severe mitral regurgitation?

    Mitral regurgitation results in a chronic volume overload on the left ventricle and ultimately results in left ventricular contractile dysfunction. Mitral valve surgery should be performed before ventricular dysfunction occurs, but there is no method to precisely predict the onset of ventricular dysfunction. Serial Doppler echocardiography studies should be performed to evaluate left ventricular size and function and assess pulmonary pressures. Surgery is recommended in the asymptomatic patient if the EF falls below 60%, or when the left ventricular end-diastolic dimension exceeds 45 mm, or if pulmonary hypertension or atrial fibrillation develops. Surgery is recommended for patients with symptoms of HF due to mitral regurgitation unless they have severe left ventricular dysfunction or other contraindications to surgery.

  • What are the choices for mitral valve surgery?

    Mitral valve replacement with mechanical or bioprosthetic valves and, more recently, mitral valve repair constitute the surgical treatments available for severe mitral regurgitation due to a primary valve abnormality. Mitral valve repair is associated with a lower surgical mortality rate and a better long-term outcome than mitral valve replacement, and is the preferred procedure when repair is possible. Mitral valve surgery is generally not indicated if the mitral regurgitation is due to left ventricular dysfunction without a primary valve abnormality.

Case 3

A 56-year-old man is seen because of progressive fatigue, dyspnea on exertion, orthopnea, and paroxysmal nocturnal dyspnea. On physical examination his blood pressure is 160/60 mm Hg. There is no jugular venous distention, but systolic pulsations of the uvula are noted, as is quick collapse of the arterial pulses, which is seen in the nail beds with gentle pressure. The lungs are clear to percussion and auscultation. There is a diffuse and hyperdynamic apex beat that is displaced laterally and inferiorly, soft first and second heart sounds, a loud third heart sound, and a grade 3/6, high-pitched, nearly holodiastolic murmur heard best at the upper left sternal border along with a grade 3/6 systolic ejection type murmur at the upper left sternal border radiating to the carotids. A late diastolic rumble is heard at the apex as well as a third heart sound.

P.66


  • What is the most likely valvular lesion in this patient?

  • What is the likely underlying cause of aortic regurgitation in this patient?

  • What is the appropriate medical therapy for the patient with aortic regurgitation?

  • When should aortic valve replacement be considered in a patient with chronic aortic regurgitation?

  • What is the surgical therapy for severe aortic regurgitation?

Case Discussion

  • What is the most likely valvular lesion in this patient?

    This patient has chronic severe aortic regurgitation. There are many physical signs to look for in the setting of aortic regurgitation, some of which are seen in this patient. These include de Musset's sign (the head bobs with each heartbeat), Corrigan's sign or waterhammer pulses, Traube's sign (booming systolic and diastolic sounds heard over the femoral arteries), Muller's sign (systolic pulsation of the uvula), Quincke's sign (capillary pulsations seen in the nail beds), and others. All of these are signs of large stroke volume and wide pulse pressure characteristic of chronic aortic regurgitation. The loud 3/6 diastolic murmur heard at the upper left sternal border is the murmur of aortic insufficiency and the systolic murmur is due to turbulent flow across the aortic valve because of the large amount of blood crossing the aortic valve. The mid-to-late diastolic rumble, namely the Austin Flint murmur, is created by rapid retrograde flow from the aorta striking the anterior mitral leaflet. Another explanation for this murmur is that the large volume of regurgitant flow partially closes the mitral valve, creating a late diastolic mitral valve gradient.

  • What is the likely underlying cause of aortic regurgitation in this patient?

    In this age-group, the most likely cause of aortic regurgitation is a bicuspid aortic valve. Causes of aortic regurgitation can be broken down into two general categories valvular disease and aortic root disease. Rheumatic heart disease, infective endocarditis, trauma, bicuspid valve, other congenital valvular defects (e.g., a fenestrated valve), systemic lupus erythematosus, rheumatoid arthritis, ankylosing spondylitis, and Whipple's disease may cause primary valvular disease. Cystic medial necrosis of the aorta (isolated or associated with Marfan's syndrome or Ehlers-Danlos syndrome), atherosclerosis, hypertension, syphilitic aortitis, and others may cause aortic root dilatation and deformity of the aortic valve, leading to inability of the valve to coapt. A dissection of the aorta may also cause aortic regurgitation by dissecting into the valve itself.

  • What is the appropriate medical therapy for the patient with aortic regurgitation?

    The use of vasodilators to delay the progression of aortic regurgitation and left ventricular dysfunction in asymptomatic patients is controversial and definite evidence of a benefit has not been demonstrated. In patients with symptoms of HF due to aortic regurgitation, vasodilators may provide symptomatic benefit but should not delay referral for aortic valve replacement.

  • P.67


  • When should aortic valve replacement be considered in a patient with chronic aortic regurgitation?

    Aortic valve surgery should be considered if the patient has symptoms of HF. In the asymptomatic patient, aortic valve replacement is recommended if the EF falls below normal or if the end-systolic dimension of the left ventricle is larger than 55 mm.

  • What is the surgical therapy for severe aortic regurgitation?

    Aortic valve replacement with a prosthetic valve is the only surgical option in most patients.

Case 4

A 32-year-old woman who recently moved to the United States from Mexico is seen because of the recent onset of palpitations associated with dyspnea on exertion, orthopnea, and paroxysmal nocturnal dyspnea with hemoptysis.

On physical examination, her blood pressure is 112/90 mm Hg and her heart rate is 120 per minute and irregularly irregular. Jugular venous distention to 10 cm H2O with a prominent V wave is noted, as are diminished arterial pulses and bibasilar rales (up to half of the lung fields bilaterally). Additional findings include a nondisplaced apex beat, a right ventricular heave palpable in the left parasternal region, a palpable pulmonic closure sound in the second left intercostal space, an accentuated S4, a loud pulmonic second sound (P2) over the left ventricular apex, a snapping sound over the left ventricular apex impulse just after the second heart sound, and a grade 3/4, low-pitched, rumbling, nearly holodiastolic murmur heard best at the cardiac apex. There is 1 to 2+ pitting edema noted in the lower extremities and presacral area.

  • What is the most likely valvular lesion in this patient?

  • What is the most common cause of mitral stenosis in adult patients?

  • What is the mortality rate associated with medically treated mitral stenosis?

  • What are the major complications of mitral stenosis?

  • What is the best treatment for symptomatic patients with mitral stenosis?

Case Discussion

  • What is the most likely valvular lesion in this patient?

    The clinical picture exhibited by this patient is characteristic of severe mitral stenosis with secondary pulmonary hypertension and cor pulmonale. The severity of the mitral stenosis is indicated by the mitral opening snap, which closely follows the second heart sound and the holodiastolic rumble. The mitral opening snap is a characteristic sign of mitral stenosis and appears to be due to a sudden tensing of the valve leaflets after the valve cusps have completed their opening excursions, and occurs shortly after (0.08 to 0.12 second) the aortic component of the second heart sound. The mitral opening snap moves closer to the second heart sound as the pressure between the left atrium and left ventricle increases. The rumbling,

    P.68


    low-pitched diastolic murmur heard at the apex is characteristic of mitral stenosis. The duration of the murmur throughout diastole indicates that there is a pressure gradient across the mitral valve throughout diastole. Pulmonary hypertension is indicated by the loud pulmonic component of the second heart sound and right ventricular heave. A P2 that can be heard at the left ventricular apex indicates pulmonary hypertension. Cor pulmonale is reflected by the elevated neck veins, and peripheral edema. The large V wave indicates tricuspid regurgitation a result of the pulmonary hypertension and cor pulmonale. Paroxysmal nocturnal dyspnea with hemoptysis is a major clue to this diagnosis and reflects a sudden increase in pulmonary capillary pressure with intraalveolar edema and hemorrhage such as might occur with exercise or new onset of atrial fibrillation.

  • What is the most common cause of mitral stenosis in adult patients?

    Mitral stenosis in adults is almost exclusively due to rheumatic heart disease. This patient described a prolonged illness at 12 years of age consistent with acute rheumatic fever but half of all patients with rheumatic mitral stenosis will not have a clear childhood history of rheumatic fever.

  • What is the mortality rate associated with medically treated mitral stenosis?

    From the time of the initial diagnosis, patients with medically treated mitral stenosis can expect a mortality rate of 20% at 5 years and 40% at 10 years. This patient faces a much less favorable prognosis because of her pulmonary hypertension and right ventricular HF. However, the risk is significantly reduced if she undergoes valve replacement, commissurotomy, or mitral balloon valvotomy.

  • What are the major complications of mitral stenosis?

    In patients with uncorrected mitral stenosis, there is a 20% lifetime risk of thromboembolism. This is often a devastating complication because the embolus most often travels to the brain, resulting in a stroke. Eighty percent of patients with systemic emboli are in atrial fibrillation. This risk is decreased by the use of anticoagulant therapy with sodium warfarin. Infective endocarditis occurs less frequently but may be a disastrous complication. Atrial fibrillation is a common complication of mitral stenosis. The left atrium is often very large due to a combination of rheumatic involvement of the atrial muscle and the high left atrial pressures, predisposing to atrial fibrillation.

  • What is the best treatment for symptomatic patients with mitral stenosis?

    Options for correction of mitral stenosis include percutaneous transvenous mitral valvuloplasty, surgical mitral commissurotomy, or mitral valve repair. In balloon valvuloplasty, the balloon is passed from the femoral vein to the right atrium, across the atrial septum, and across the mitral valve. The balloon is inflated, cracking open the valve. This is the preferred procedure in experienced hands if the valve anatomy is favorable and there are no contraindications. Results are also good with surgical commissurotomy or mitral valve replacement.

Suggested Readings

Bonow RO, Lakatos E, Maron BJ, etal. Serial long-term assessment of the natural history of asymptomatic patients with chronic aortic regurgitation and normal left ventricular systolic function. Circulation 1991;84:1625 1635.

P.69


Carabello BA. Concentric vs eccentric remodeling. J Card Fail 2002;8:S258 S263.

Carabello BA. Modern management of mitral stenosis. Circulation 2005;112:432 437.

Carabello BA. Vasodilators in aortic regurgitation-where is the evidence of their effectiveness? N Engl J Med 2005;353:1400 1402.

Freeman RV, Otto CM. Spectrum of calcific aortic valve disease: pathogenesis, disease progression, and treatment strategies. Circulation 2005;111:3316 3326.

Goldsmith I, Turpie AG, Lip GY. Valvular heart disease and prosthetic heart valves. Br Med J 2002;325:1228 1231.

Lambo NJ, Dell'Italia LJ, Crawford MH, etal. Bedside diagnosis of systolic murmurs. N Engl J Med 1988;318:1572 1579.

Lieberman EG, Bashore TM, Hermiller JB, etal. Balloon aortic valvuloplasty in adults: failure of procedure to improve long-term survival. J Am Coll Cardiol 1995;26:1522 1528.

Pellikka PA, Nishimura RA, Bailey KR, etal. The natural history of adults with asymptomatic, hemodynamically significant aortic stenosis. J Am Coll Cardiol 1990;15:1012 1017.

Reyes VP, Raju BS, Wynne J, etal. Percutaneous balloon valvuloplasty compared with open surgical commissurotomy for mitral stenosis. N Engl J Med 1994;331:961 967.

Rozich JD, Carabello BA, Usher BW, etal. Mitral valve replacement with and without chordal preservation in patients with chronic mitral regurgitation: mechanisms for differences in postoperative ejection performance. Circulation 1992;86:1718 1726.

Vongpastanasin W, Hills LD, Lange RA. Medical progress: prosthetic heart valves. N Engl J Med 1996;335:407 416.



Internal Medicine Casebook. Real Patients, Real Answers
The Internal Medicine Casebook: Real Patients, Real Answers
ISBN: 0781765293
EAN: 2147483647
Year: 2007
Pages: 14

flylib.com © 2008-2017.
If you may any questions please contact us: flylib@qtcs.net